You are on page 1of 158

Master of Business Administration- MBA Semester 1 MB0038 Management Process and Organizational Behaviour - 4 Credits Assignment Set- 1 (60

Marks) Q1. State the characteristics of management. Ans:- Management is a distinct activity having the following salient features or characteristics: 1. Goal-oriented: Management is a purposeful activity. It co-ordinates the efforts of employees to achieve the goals of the organization. The success of management is measured by the extent to which the organizational goals are achieved. It is imperative that the organizational goals must be well-defined and properly understood by the mangers at various levels. 2. Economic Resource: Management is one of the factors of production together with land, labour and capital. It is the most critical input in the success of any organized group activity. It is the force which assembles and integrates other resources, namely, labour, capital and materials. These factors do not by themselves ensure production, they require the catalyst of management to produce goods and services required by the society. Thus, management is an essential ingredient of an organization. 3. Distinct Process: Management is a distinct process consisting of such functions as planning, organizing, staffing, directing and controlling. These functions are so interwoven that it is not possible to lay down exactly the sequence of various functions or their relative significance. In essence, the process of management involves decision-making and putting of decisions into practice. 4. Integrative Force: The essence of management is integration of human and other resources to achieve the desired objectives. All these resources are made available to those who manage. Managers apply knowledge, experience and management principles for getting the results from the workers by the use of non-human resources. Managers also seek to harmonize the individuals goals with the organizational goals for the smooth working of the organization.

5. Intangible Force: Management has been called an unseen force. Its presence is evidenced by the result of its efforts-orderliness, informed employees, buoyant spirit and adequate work output. Thus, feeling of management is result-oriented. One may not see with the naked eyes the functioning of management but its results are apparently known. People often remark of the effectiveness (or ineffectiveness) of management on the basis of the end results, although they cant observe it during operation. 6. Results through Others: The managers cannot do everything themselves. They must have the necessary ability and skills to get work accomplished through the efforts of others. They must motivate the subordinates for the accomplishment of the tasks assigned to them. 7. A Science and an Art: Management has an organized body of knowledge consisting of welldefined concepts, principles and techniques which have wide applications. So it is treated as a science. The application of these concepts, principles and techniques requires specialized knowledge and skills on the part of the manager. Since the skills acquired by a manager are his personal possession, management is viewed as an art. 8. System of Authority: Management as a team of managers represents a system of authority, a hierarchy of command and control. Managers at different levels possess varying degrees of authority. Generally, as we move down in the managerial hierarchy, the degree of authority gets gradually reduced. Authority enables the managers to perform their functions effectively. 9. Multi-disciplinary Subject: Management has grown as a field of study (i.e. discipline) taking the help of so many other disciplines such as Engineering, Anthropology, Sociology and Psychology. Much of the management literature is the result of association of these disciplines. For instance, productivity orientation drew its inspiration from Industrial Engineering and human relations orientation from Psychology. Similarly, Sociology and Operations Research have also contributed to the development of management science. 10. Universal Application: Management is universal in character. The principles and techniques of management are equally applicable in the fields of business, education, military, government and hospital. Henri Fayol suggested that principles of management would apply more or less i

Q2. What are the 14 principles of management of Henri Fayol? Ans:- Henri Fayol, a mining engineer and manager by profession, defined the nature and working patterns of the twentieth-century organization in his book, General and Industrial Management, published in 1916. In it, he laid down what he called 14 principles of management. This theory is also called the Administrative Theory. The principles of the theory are: 1. Division of work: tasks should be divided up with employees specializing in a limited set of tasks so that expertise is developed and productivity increased. 2. Authority and responsibility: authority is the right to give orders and entails enforcing them with rewards and penalties; authority should be matched with corresponding responsibility. 3. Discipline: this is essential for the smooth running of business and is dependent on good leadership, clear and fair arguments, and the judicious application of penalties. 4. Unity of command: for any action whatsoever, an employee should receive orders from one superior only; otherwise authority, discipline, order, and stability are threatened. 5. Unity of direction: a group of activities concerned with a single objective should be cocoordinated by a single plan under one head. 6. Subordination of individual interest to general interest: individual or group goals must not be allowed to override those of the business. 7. Remuneration of personnel: this may be achieved by various methods but it should be fair, encourage effort, and not lead to overpayment. 8. Centralization: the extent to which orders should be issued only from the top of the organization is a problem which should take into account its characteristics, such as size and the capabilities of the personnel. 9. Scalar chain (line of authority): communications should normally flow up and down the line of authority running from the top to the bottom of the organization, but sideways communication

between those of equivalent rank in different departments can be desirable so long as superiors are kept informed. 10. Order: both materials and personnel must always be in their proper place; people must be suited to their posts so there must be careful organization of work and selection of personnel. 11. Equity: personnel must be treated with kindness and justice. 12. Stability of tenure of personnel: rapid turnover of personnel should be avoided because of the time required for the development of expertise. 13. Initiative: all employees should be encouraged to exercise initiative within limits imposed by the requirements of authority and discipline. 14. Esprit de corps: efforts must be made to promote harmony within the organization and prevent dissension and divisiveness. Q3. Distinguish between internal and external forces of change. Ans:- Internal forces Any change in organizations internal factors may also necessitate change. Such a change is required because of two reasons: change in managerial personnel and deficiency in existing organizational practices. Change in the top management: Change in the top management and consequent change in the ideas to run the organization also leads to change in the system, structure and processes. Old managers are replaced by new managers which are necessitated because of retirement, promotion, transfer or dismissal. Each new manager brings his own ideas and way of working into the organization. The formal or informal relationships may change because of changes in top management. Moreover, attitudes, ideology, leadership style of the person may be different from the earlier one, this will reflect in their actions and decisions. The result is that an organization has to change accordingly.

Change in size of the organization: Change in the organizations size leads to change in the internal structure and complexity of the operations in the organization. Performance gaps: When a gap between set target and actual results (in terms of market share, employee productivity and profit) is identified, organizations face the forces to change and reduce the gap. Employee needs and values: With changing needs and values of the employees, organizations change their policies. For example, attractive financial incentives, challenging assignments, vertical growth opportunities and autonomy at work may be provided in an organization to attract and retain its effective employees. Deficiency in existing organization: Sometimes, changes are necessary because of deficiency in the present organizational arrangement and process. These deficiencies may be in the form of unmanageable span of management, large number of managerial levels, lack of co-ordination between various departments, obstacles in communication, multiplicity of committees, lack of uniformity in policy decisions, lack of co-operation between line and staff and so on. External forces Each organization has goals and responsibility related to others in its environment. Thus, an organization must not only deal with its environment in conducting its affairs, but also give consideration to the goals of others, as it establishes its goals and conducts its operations. The present-day environment is dynamic and will continue to be dynamic. Changes in social, political, economic, technological, and legal environment force organizations to change themselves. Such changes may result in organizational changes like major functions, production process, labour-management relations, nature of competition, economic constraints, organization methods, etc. In order to survive in the changing environment, organization must change. Technology: Technological changes are responsible for changing the nature of the job performed at all levels in an organization. When there is a change in technology in the organizations environment and other organizations adopt the new technology, the organization under focus becomes less cost-effective and its competitive position weakens. Therefore, it has to adopt new

technology. When the organizations adopt a new technology, its work structures are affected and a new equilibrium has to be established. We have seen that technology has impact on organization structure, organizational processes, and behaviour of people. For example, computers and automation have made significant impact on organizational functioning. Business scenario: Due to rapid changes in the business scenario with increasing competition and global economy, the needs and demands are also changing among the customers, suppliers and other stakeholders. Organizations are, therefore, forced to change their operational methods to meet the demands of the stakeholders. Since every organization exports its outputs to the environment, an organization has to face competition in the market. There may be two types of forces which may affect the competitive position of an organization other organizations supplying the same products and, buyers who are buying the product. Any change in these forces may require suitable changes in the organization. For example, when Indian economy was liberalized (the process still continues), there were many foreign organizations which entered the Indian market. This forced many Indian organizations to re-align themselves with the new situation. The result is that there have been many cases of divesting the business and concentrating on the core business, acquiring core business, and developing competitive competence to face competitive threats. Similarly, there may be changes in buyers in terms of their needs, liking-disliking, and income disposal for a product. These changes force the organizations to bring those products which meet buyers requirements. Environmental and National factors: Environmental factors such as economic, political and demographic and legal factors play a vital role in devising organizational policies and strategy. Any change in these political and legal factors may affect the organizational operation. For example, organizations may have to change their employment policies in accordance with the government policy, demand of the non-government organizations and changing economic conditions of a country. Social changes: Social changes reflect in terms of peoples aspirations, their needs, and their way of working. Social changes have taken place because of the several forces like level of education, urbanization, feeling of autonomy, and international impact due to new information

sources. These social changes affect the behaviour of people in the organization. Therefore it is required to make adjustment in its working so that it matches with people. Q4. Define emotional intelligence. Explain Golemans model of emotional intelligence. Ans;Emotional Intelligence - EI - is a relatively recent behavioural model, rising to

prominence with Daniel Golemans 1995 Book called Emotional Intelligence. The early Emotional Intelligence theory was originally developed during the 1970s and 80s by the work and writings of psychologists Howard Gardner (Harvard), Peter Salovey (Yale) and John Jack Mayer (New Hampshire). Emotional Intelligence is increasingly relevant to organizational development and developing people, because the EI principles provide a new way to understand and assess peoples behaviours, management styles, attitudes, interpersonal skills, and potential. Emotional Intelligence is an important consideration in human resources planning, job profiling, recruitment interviewing and selection, management development, customer relations and customer service, and more. Ever since the publication of Daniel Golemans first book on the topic in 1995, emotional intelligence has become one of the hottest buzzwords in corporate America. For instance, when the Harvard Business Review published an article on the topic two years ago, it attracted a higher percentage of readers than any other article published in that periodical in the last 40 years. When the CEO of Johnson & Johnson read that article, he was so impressed that he had copies sent out to the 400 top executives in the company worldwide. "Emotional intelligence is the capacity for recognizing our own feelings and those of others, for motivating ourselves, for managing emotions well in ourselves and in our relationships." (Snow, 2001) Golemans Model of Emotional Intelligence Daniel Goleman and the Hay Group have identified a set of competencies that differentiate individuals with Emotional Intelligence. The competencies fall into four clusters:

Self-Awareness: Capacity for understanding one's emotions, one's strengths, and one's weaknesses. Self-Management: Capacity for effectively managing one's motives and regulating one's behavior. Social Awareness: Capacity for understanding what others are saying and feeling and why they feel and act as they do. Relationship Management: Capacity for acting in such a way that one is able to get desired results from others and reach personal goals.

The most popular and accepted mixed model of emotional intelligence is the one proposed by Goleman (1995). He viewed emotional intelligence as a total of personal and social competences. Personal competence determines how we manage ourselves, whereas social competence determines how we handle our interpersonal relationships. Q5. Explain the different leadership styles as per Managerial Leadership Grid Theory Ans:- The Managerial and Leadership Grid The Ohio studies led to two dimensions of leadership behaviour-concern for tasks and concern for relations. Almost in the same style, the Michigan University studies made the distinction between job-centred and production- centred leaders. Blake and Mouton rated these concepts in a framework called the Managerial Grid. They interpreted the concepts in a broad way. Blake and Mouton have used "Concern for Production" and "Concern for People" in their Managerial Grid on horizontal and vertical axes respectively. Managers may be concerned for their people and they also must also have some concern for the work to be done. The question is, how much attention do they pay to one or the other? This is a model defined by Blake and Mouton in the early 1960s.It included Impoverished management Authority-compliance Country Club management Middle of the road management

Team management

The Managerial Grid was the original name; the modifications were made by Robert R Blake and Anne Adams McCanse. After the modifications it was named as Leadership Grid.

Leadership Grid an approach to understanding a leaders concern for results (production) and concern for people 1. The impoverished style (1, 1). The indifferent Leader (Evade & Elude) In this style, managers have low concern for both people and production. Managers use this style to avoid getting into trouble. The main concern for the manager is not to be held responsible for any mistakes, which results in less innovative decisions. A leader uses a "delegate and disappear" management style. Since they are not committed to either task accomplishment or maintenance; they essentially allow their team to do whatever it wishes and prefer to detach themselves from the team process by allowing the team to suffer from a series of power struggles. 2. The country club style (1, 9). The accommodating Leader (Yield & Comply) This style has a high concern for people and a low concern for production. Managers using this style pay much attention to the security and comfort of the employees, in hopes that this would increase performance. The resulting atmosphere is usually friendly, but not necessarily that productive. This person uses predominantly reward power to maintain discipline and to encourage the team to accomplish its goals. Conversely, they are almost incapable of employing the more punitive

coercive and legitimate powers. This inability results from fear that using such powers could jeopardize relationships with the other team members. 3. The produce or perish style (9, 1). The Controlling Leader (Direct & Dominate) This believes in the authority-obedience. With a high concern for production, and a low concern for people, managers using this style find employee needs unimportant; they provide their employees with money and expect performance back. Managers using this style also pressure their employees through rules and punishments to achieve the company goals. This dictatorial style is based on Theory X of Douglas McGregor, and is commonly applied by companies on the edge of real or perceived failure. This is used in case of crisis management. People who get this rating are very much task-oriented and are hard on their workers (autocratic). There is little or no allowance for co-operation or collaboration. Heavily task-oriented people display these characteristics: they are very strong on schedules; they expect people to do what they are told without question or debate; when something goes wrong they tend to focus on who is to blame rather than concentrate on exactly what is wrong and how to prevent it; they are intolerant of what they see as dissent (it may just be someones creativity), so it is difficult for their subordinates to contribute or develop. 4. The middle-of-the-road style (5, 5). The Status Quo Leader. (Balance & Compromise) It is Organization - man management approach,which believes that the adequate organization performance is possible through balancing the necessity to get out wprk with maintaining morale of people at satisfactory level. Managers using this style try to balance between company goals and workers needs. By giving some concern to both people and production, managers who use this style hope to achieve acceptable performance. 5. The team style (9, 9). The Sound / Team Leader (Contribute & Commit) This is based on the aspect that work accomplishment is from committed people; interdependence through a common stake in the organization purpose leads to relationships of trust and respect. In this style, high concern is paid both to people and production. As suggested

by the propositions of Theory Y, managers choosing to use this style encourage teamwork and commitment among employees. This method relies heavily on making employees feel as a constructive part of the company. This type of person leads by positive example and endeavours to foster a team environment in which all team members can reach their highest potential, both as team members and as people. They encourage the team to reach team goals as effectively as possible, while also working tirelessly to strengthen the bonds among the various members. They normally form and lead some of the most productive teams. Q6 . Mr. Suresh Kumar is the VP- HR of a leading Financial services company. He is having a meeting with Ms. Rejani Chandran leading HR consultant. Mr. Suresh is concerned about creating an environment that helps in increasing the job satisfaction amongst employees. Assume that you are Ms. Rejani, the HR consultant. What suggestions you will give to Mr. Suresh, for creating an environment that increases job satisfaction?

Ans:- Job satisfaction is the sense of fulfillment and pride felt by people who enjoy their work and do it well. For an organization, satisfied work force ensures commitment to high quality performance and increased productivity Job satisfaction helps organizations to reduce complaints and grievances, absenteeism, turnover, and termination. Job satisfaction is also linked to a healthier work force and has been found to be a good indicator of longevity. And although only little correlation has been found between job satisfaction and productivity, it has also been found that satisfying or delighting employees is a prerequisite to satisfying or delighting customers, thus protecting the bottom line (Brown, 1996). The following suggestions can help a worker find personal job satisfaction:

1. Seek opportunities to demonstrate skills and talents. 2. Develop communication skills. 3. Acquire job related skills and try to implement them. 4. Demonstrate creativity and initiative. 5. Improve team building and leadership skill. 6. Learn to de-stress.

Master of Business Administration- MBA Semester 1 MB0038 Management Process and Organizational Behaviour - 4 Credits Assignment Set- 2 (60 Marks) Q1. Explain sensitivity training. Ans:- Sensitivity training (also known as T-group, T standing for training). This approach evolved from the group dynamics concept of Kurt Lewin and the first sensitivity training session was held in 1946 in State Teachers College, New Britain, USA. Since then, it spread to numerous training centers in USA and other countries. Sensitivity training is a smallgroup interaction process in the unstructured form which requires people to become sensitive to others feelings in order to develop reasonable group activity. The objectives of sensitivity training are as follows: 1. To make participants increasingly aware of, and sensitive to, the emotional reactions and expressions in themselves and others. 2. To increase the ability of participants to perceive, and to learn from, the consequences of their actions through attention to their own and others feelings. 3. To stimulate the clarification and development of personal values and goals consonant with a democratic and scientific approach to problems of personal and social decisions and actions. 4. To develop achievement of behavioural effectiveness in participants. 5. To develop concepts and theoretical framework for linking personal values and goals to actions consistent with these inner factors and situational requirements. Process of Sensitivity Training Sensitivity training focuses on small group (T-group) with number of members ranging from ten to twelve. T-groups are designed to provide members with experiential learning about group dynamics, leadership and interpersonal relationships. The basic T-group training or sensitivity training is to change the standards, attitudes and behavior of individuals by using psychological

techniques and programs. Based on the sources from where these members are drawn, there may be three types of T-group: stranger-lab, cousin-lab, and family-lab. In the stranger-lab, all participants are from different organizations and they are strangers to each other. In cousin-lab, all participants are from the same organization but from different units. Q2. Describe the bases of power. Ans:- Bases of Power Power can be categorized into two types: Formal and informal A. Formal Power: It is based on the position of an individual in an organization. Formal power is derived from either ones ability to coerce or reward others or is derived from the formal authority vested in the individual due to his/ her strategic position in the organizational hierarchy. For example, a manager may threaten to withhold a pay raise, or to transfer, demote, or even recommend the firing of a subordinate who does not act as desired. Such coercive power is the extent to which a manager can deny desired rewards or administer punishments to control other people. The availability of coercive power also varies across organizations. The presence of unions and organizational policies on employee treatment can weaken this power base significantly. Formal power may be categorized into four types which are as follows: 1. Coercive Power: The coercive power base is being dependent on fear. It is based on the application, or the threat of application, of physical sanctions such as the infliction of pain, the generation of frustration through restriction of movement, or the controlling by force of basic physiological or safety needs. In an organization one can exercise power over another if they have the power to dismiss, suspend, demote another assuming that the job is valuable to the person on whom power is being unleashed. 2. Reward Power: The opposite of coercive power is reward power. Reward power is the extent to which a manager can use extrinsic and intrinsic rewards to control other people. Examples of such rewards include money, promotions, compliments, or enriched jobs. Although all managers

have some access to rewards, success in accessing and utilizing rewards to achieve influence varies according to the skills of the manager. 3. Legitimate Power: The third base of position power is legitimate power, or formal authority .It stems from the extent to which a manager can use subordinates internalized values or beliefs that the boss has a right of command to control their behavior. For example, the boss may have the formal authority to approve or deny such employee requests as job transfers, equipment purchases, personal time off, or overtime work. Legitimate power represents a special kind of power a manager has because subordinates believe it is legitimate for a person occupying the managerial position to have the right to command. The lack of this is legitimacy will result in authority not being accepted by subordinates. 4. Information Power: This type of power is derived from access to and control over information. When people have needed information, others become dependant on them. (For example, managers have access to data that subordinates do not have). Normally the higher the level, the more information would be accessed by managers. B. Personal Power Personal power resides in the individual and is independent of that individuals position. Three bases of personal power are expertise, rational persuasion, and reference. Expert power is the ability to control another persons behavior by virtue of possessing knowledge, experience, or judgment that the other person lacks, but needs. A subordinate obeys a supervisor possessing expert power because the boss ordinarily knows more about what is to be done or how it is to be done than does the subordinate. Expert power is relative, not absolute. However the table may turn in case the subordinate has superior knowledge or skills than his/ her boss. In this age of technology driven environments, the second proposition holds true in many occasions where the boss is dependent heavily on the juniors for technologically oriented support. Rational persuasion is the ability to control anothers behavior, since, through the individuals efforts, the person accepts the desirability of an offered goal and a viable way of achieving it.

Rational persuasion involves both explaining the desirability of expected outcomes and showing how specific actions will achieve these outcomes. Referent power is the ability to control anothers behavior because the person wants to identify with the power source. In this case, a subordinate obeys the boss because he or she wants to behave, perceive, or believe as the boss does. This obedience may occur, for example, because the subordinate likes the boss personally and therefore tries to do things the way the boss wants them done. In a sense, the subordinate attempts to avoid doing anything that would interfere with the pleasing boss subordinate relationship. Followership is not based on what the subordinate will get for specific actions or specific levels of performance, but on what the individual represents a path toward lucrative future prospects. Charismatic Power is an extension of referent power stemming from an individuals personality and interpersonal style. Others follow because they can articulate attractive visions, take personal risks, demonstrate follower sensitivity, etc. Q3. What are the hindrances that we face in perception?

Ans:- Following are the barriers to perception: a) Selective perception - People selectively interpret what they see on the basis of their interests, background, experience, knowledge, exposure, and attitudes. The tendency to see what we want to see using short cuts can make us draw unwarranted conclusions from an ambiguous situation. b) Halo Effect This effect occurs when we draw a general impression on the basis of a single characteristic. But what this experiment demonstrates is that although we can understand the halo effect intellectually, we often have no idea when it is actually happening. This is what makes it such a useful effect for marketers and politicians. We quite naturally make the kinds of adjustments demonstrated in this experiment without even realising it. And then, even when it's pointed out to us, we may well still deny it.

c) Contrast Effect Individuals do not evaluate a person in isolation. Their reaction to one person is influenced by other people they have encountered recently. d) Projection This tendency to attribute one`s own characteristics to other people is called projection. This too can distort perceptions made about others. When managers engage in projection, they compromise their ability to respond to individual differences. They tend to see people as more homogeneous than they really are. e) Stereotyping Judging someone on the basis of our perception of the group to which he or she belongs. Stereotypes are generalizations about a group of people whereby we attribute a defined set of characteristics to this group. These classifications can be positive or negative, such as when various nationalities are stereotyped as friendly or unfriendly. It is easier to create stereotypes when there is a clearly visible and consistent attribute that can easily be recognized. This is why people of colour, police and women are so easily stereotyped. People from stereotyped groups can find this very disturbing as they experience an apprehension (stereotype threat) of being treated unfairly. f) First-Impression Error - is the tendency to form lasting opinions about an individual based on initial perceptions. We tend to remember what we perceive first about a person and sometimes we are quite reluctant to change our initial impressions. Q4. What are the consequences of conflict in organisations? Ans:- The consequences of conflict can be positive or negative, as shown below:

Q5. Explain the four processes of Social Learning Theory. Ans:- The social learning theory was proposed by Bandura. It recognizes the importance of observing and modeling the behaviors, attitudes, and emotional reactions of others. According to Bandura (1977), most human behavior is learned observationally through modeling: from observing others one forms an idea of how new behaviors are performed, and on later occasions this coded information serves as a guide for action. Social learning theory explains human behavior in terms of continuous reciprocal interaction between cognitive, behavioral, and environmental influences. Social learning has four processes: 1. Attention processes People learn from a model only when they recognize and pay attention to its critical features. In order to learn, it is required to pay attention. Anything that detracts the attention is going to have a negative effect on observational learning. If the is model interesting or there is a novel aspect to the situation, it is more likely to dedicate the full attention to learning. 2. Retention processes A models influence will depend on how well the individual remembers the models action after the it is no longer readily available. The ability to store information is also an important part of the learning process. Retention can be affected by a number of factors, but the ability to pull up information later and act on it is vital to observational learning. 3. Motor reproduction processes After a person has seen a new behavior by observing the model, the watching must be converted to doing. The ability to store information is also an important part of the learning process. Retention can be affected by a number of factors, but the ability to pull up information later and act on it is vital to observational learning. 4. Reinforcement processes Individuals will be motivated to exhibit the modeled behavior if positive incentives or rewards are provided. Finally, in order for observational learning to be successful, you have to be motivated to imitate the behavior that has been modeled. Reinforcement and punishment play an important role in motivation. While experiencing these

motivators can be highly effective, so can observing other experience some type of reinforcement or punishment. For example, if you see another student rewarded with extra credit for being to class on time, you might start to show up a few minutes early each day. Q6. Ms.Chanchal Das Gupta is a recruitment specialist. For the post of QC Manager, she interviews three candidates. Given below are the physical characteristics of the candidates. Candidate Mr.Ravi Mr.Gineesh Physical Characteristics Muscular, thick skin, rectangular shaped. Thin, delicate build, large brain, tall. Soft, round shaped, underdeveloped

Mr.Ramgopal muscles.

From the above descriptions, what personality traits can Ms. Chanchal derive out of the candidates as per Sheldons theory of personality? Ans:- William Sheldon (1940, 1942, cited in Phares, 1991) classified personality according to body type. He called this a persons somatotype. Sheldon identified three main somatotypes: Sheldons Classification of Personality Types.

As per Sheldon`s theory of personality, below are the traits that Ms. Chanchal can derive:

Mr. Ravi represents Mesomorph body type. He is well-proportioned. Psychologically he is Adventurous, Courageous, Indifferent to what others think or want, Assertive/bold, Zest for physical activity, Competitive, With a desire for power/dominance, And a love of risk/chance

Mr. Gineesh represents Ectomorph body type. Psychologically he is Self-conscious, Private, Introverted, Inhibited, Socially anxious, Artistic, Intense, Emotionally restrained, Thoughtful

Mr. Ramgopal represents Endomorph body type. Psychologically he is Sociable, Funloving, Love of food, Tolerant, Even-tempered, Good humoured, Relaxed, With a love of comfort, And has a need for affection.

Master of Business Administration Semester I MB0039 Business Communication - 4 Credits Assignment Set- 1 (60 Marks) Q1.List the importance of effective communication in the workplace Ans:- The Importance of Communication in the Workplace Communication is the nerve center of business today. As you go up the corporate ladder, you will find that communication skills are required, more than technical skills. Communication research has revealed that among the factors most important for managerial success, communication skills rank above technical skills. Several surveys conducted among people who have been successful in their professions have indicated that communication skills are more vital to job success than subjects taken in college. Communication has assumed even greater importance today, since the new model of business is based on teamwork, rather than on individual action. Teamwork requires greater coordination and communication. Communication is also required all the more in this age of information and technology. Without communication and human skills, technology will overwhelm an organization. Communication helps to make sense of technology and to manage all this information. For example, communication is required to explain a new computer program or software. While computers can perform routine tasks, jobs like responding to customers needs require a high degree of communication skills. Effective communication serves the following specific purposes in an organization Greater Awareness of Organizational Goals and Teamwork When there is open communication between superiors, co-workers and subordinates, there is smooth flow of information regarding the goals of the organization. Coordination between the different departments in particular, leads to greater motivation to work together towards achieving a common organizational goal, rather than working in isolation.

Better Employer-employee Relationships By listening to employees, showing empathy and giving them the freedom to express their opinions without fear of being repressed, a manager can create a climate of openness that leads to better work relationships. Employees will then feel more comfortable in approaching their superiors and discussing any matter with them.

Problem-solving Effective communication can help resolve conflicts between coworkers, work related and performance related problems. Faceto-face communication is especially suited for achieving this task, since it is one to one and highly personalized in nature.

Improved Performance Effective communication by managers at the time of appraising the performance of their employees can point out areas for improvement. A constructive review of performance, through which a manager gives positive feedback and counsels the employee, instead of criticizing him for poor performance, can motivate the employee to perform better.

Stronger Link between Managers and the External Environment Apart from internal communication within the organization, effective communication by managers with external audiences such as customers, government, bankers, media and suppliers leads to a better rapport with them.A manager will be able to understand the needs of his customers, be aware of the presence of quality suppliers of material, of government regulations and of the expectations of the community at large, only through proper communication.

Q2. Explain the different aspects of non-verbal communication Ans:- Different aspects of non-verbal communication in detail1. Kinesics This is the most often studied and important area of non-verbal communication and refers to body movements of any kind. Different body movements can express inner states of emotion.

Facial Expressions can convey feelings of surprise, happiness, anger and sadness. If you meet a long lost friend and say Im very happy to meet you again, but with a sad facial expression, it conveys the exact opposite meaning. Eye Movements, such as wide open pupils express feelings of surprise, excitement or even fear. The importance of eye contact with ones audience was pointed out earlier. Direct eye contact is an indication of intensity and interest, while lack of it can convey feelings of nervousness and guilt. Gestures, such as movement of the hands while giving a lecture or presentation indicates a high level of involvement in what you are saying. On the other hand, shuffling of the feet is a sign of nervousness and speaking with ones hands in ones pockets is considered to be casual or even rude. Head Movements like nodding the head can convey interest, appreciation, agreement or understanding. Body Shape and Posture Body shape is not within ones control but can be stereotyped to convey certain meanings. For example, someone who is strong and muscular is generally thought to be athletic, as opposed to a person who is short and fat! Posture on the other hand is within our control. In formal settings such as job interviews or classroom settings, it is essential that you maintain an erect posture to convey that you are attentive, since slouching or a relaxed posture conveys a casual attitude. Physical Appearance Our outward appearance, including the way we dress and the jewelry and make-up that we wear can convey an impression of formality or informality. Going to a job interview dressed in blue jeans or not sticking to a stipulated dress code at the workplace can convey that you are a rebel, non-conformist or a very casual person. Therefore, it is important to take care of your appearance, so that you convey the right meaning to others.

2. Proxemics Proxemics is derived from the word proximity or closeness and is the communication term for personal space and distance. The space and distance which we choose to keep from people is also part of non-verbal communication. Each of us has our own inner and outer circles, which differ for different people. Our inner most circle is an intimate space, into which we generally admit only select people such as family and close friends. Next comes a personal space which might include other friends and colleagues or coworkers. These two spaces involve communication of an informal nature. Most of us also have a social and public space, which includes official or workplace relationships, where the communication is of a more formal nature. In a business context, it is more relevant to understand the concept of fixed space and semifixed space. Fixed space means that the physical features of the work environment such as furniture, room size and seating arrangement are permanent. This conveys an impression of formality. On the other hand, semi-fixed space means that certain elements of the environment can be changed for example, the seating arrangement could be changed and this conveys an impression of informality. Sometimes, use of space at the workplace can determine leadership positions. For example, seating at the head of the table conveys leadership or authority. A round table meeting, however, conveys the idea of equality, since no one can be seated at the head of the table! All points of a circle are the same. That is why when heads of state meet (as in UN Security Council meetings), it is always a round table discussion, since all heads are equal. Space should therefore be used carefully in a work environment, so as to convey the right impressions.

3. Time Language This refers to the meaning or importance attached to time and varies between different people. One person may value time more than another. Similarly, time language also varies across cultures. In most western cultures for example, punctuality is considered to be important. Arriving late for a business meeting is inexcusable. In other cultures, it is more relaxed and time is not given that much importance. We convey messages to others through the time we spend on a work related activity or by the importance that we give to time. Arriving early at work or for a job interview shows interest, involvement and seriousness. Spending time with an employee and giving him suggestions on how to improve his performance shows interest and involvement in his career growth. 4. Paralanguage Para means like or similar to, therefore paralanguage means like language. Of all the forms of non-verbal communication, paralanguage is closest to verbal communication. It refers to the tone of voice with which something is said. In other words, it is how something is said, and not what is said. The tone of voice includes the pitch (high or low pitch), the pace (slow or fast) the emphasis on words and the volume (soft or loud) and can convey different moods and emotions, as mentioned earlier in this unit. Example: The statement I practice good business communication can be understood in different ways, depending on the emphasis on certain words. Saying I practice good business communication means that I alone practice it above anyone else. On the other hand, saying I practice good business communication could be interpreted to mean that you communicate particularly well in a business context, rather than in a general context. The important point to keep in mind regarding tone of voice is to avoid mixed signals that is, making sure that what you say is consistent with how you say it. 5. Physical Context This refers to the physical environment or surroundings within which we communicate and includes two aspects 1) color and layout and 2) design.

Colors are known for their symbolic meaning and have associations with different feelings. For example, colors like black and grey are associated with death, mourning and negative feelings. Yellow and green are associated with more positive feelings. Of course, these can also vary across cultures. The point to remember is that you can make the right impressions with use of the right colors. Layout in a work environment refers to the size of an office, or the arrangement of furniture. Design refers to the type of chairs, desks or carpeting. All these can convey status, formality or informality. We have seen how the types of non-verbal communication outnumber the types of verbal communication. Non-verbal communication is an important supplement to verbal

communication and can enhance verbal communication, if used in a positive way. The sender should use the right non-verbal cues to convey a positive message, while the receiver should learn to look for unintended messages conveyed by non-verbal communication. Q3. Write short notes on (a) Upward communication (b) Downward communication (c) Horizontal communication Ans:(a) Upward Communication This may be defined as information that flows from subordinates to superiors. Some of the reasons for upward communication include discussing work related problems, giving suggestions for improvement and sharing feelings about the job and co-workers. This type of communication has both benefits and disadvantages. One of the biggest benefits is problem-solving. Once a subordinate has brought a problem to his superiors notice, chances are that the problem will not recur, since the subordinate learns from his superior how to tackle it the next time. Thus, his ability to solve new problems and therefore his managerial ability, improves. Another benefit that could arise from upward communication is that valuable ideas and

suggestions may sometimes come from lower level employees. Therefore organizations should encourage this kind of communication. A third benefit is that employees learn to accept the decisions of management and thereby work as a team. The biggest problem associated with this type of communication is that it may lead to handing down of decisions by superiors. When subordinates frequently seek the superiors guidance, the latter may adopt an authoritarian approach and merely give instructions, disregarding the subordinates opinion completely. (b) Downward Communication This may be defined as information that flows from superiors to subordinates. The most common reasons for downward communication are for giving job instructions, explaining company rules, policies and procedures and giving feedback regarding job performance. A number of studies have indicated that regular downward communication in the form of feedback given to employees is the most important factor affecting job satisfaction. Therefore organizations today are trying to encourage more of this type of communication. There are both benefits and disadvantages associated with this type of communication. Downward communication that provides regular feedback will be beneficial if the feedback or review of performance is constructive. A constructive review is one where a manager counsels an employee, or advises him on how to improve his performance. On the other hand, a destructive review can destroy employee morale and confidence. Regular downward communication also creates a climate of transparency or openness, where information is passed on through official channels, rather than through rumors. Thirdly, downward communication boosts employee morale, since it indicates that management is involved in their progress.

The problems with this type of communication are the danger of doing destructive reviews, as mentioned, and that of message overload. This means that superiors many sometimes burden their subordinates with too many instructions, leading to confusion. (c) Horizontal Communication This type of communication is also known as lateral communication. It may be defined as communication that takes place between co-workers in the same department, or in different departments, with different areas of responsibility. For example, Sales Managers and Advertising Managers in the Marketing department, or Marketing Managers and Finance Managers. The reasons for this type of communication are for coordination of tasks, sharing of information regarding goals of the organization, resolving interpersonal or work related problems and building rapport. The biggest potential benefit of horizontal communication is the sense of teamwork that is created. Regular communication of this type ensures that all co-workers work together towards achieving a common goal in the overall interest of the organization. The biggest potential problem is that conflicts such as ego clashes are bound to arise, when co-workers at the same level communicate on a regular basis. In spite of these problems, horizontal or lateral communication has become more important in todays business scenario than upward or downward communication. This is because the organizational pyramid indicating the different hierarchies or levels in an organization has flattened. This is illustrated by the diagrams given below.

The first diagram illustrates the previous organizational pyramid which was a multi-layer pyramid. In this type of pyramid, vertical, i.e., upward and downward communication still plays an important role. This is still the case in many traditionally run organizations today. However, this has been replaced by a compressed or flattened pyramid where the hierarchy has diminished, as shown in the second diagram. Thanks to technology and computers which help in faster decision making, a manager today has a larger span of control. He or she can supervise and control more number of people than before. This in turn has led to greater empowerment, which means that even lower level employees are now being given decision making authority. Therefore, in the absence of several layers, there is greater lateral communication than before.

Q4.Explain the different barriers to listening .List the differences between discriminative listening and comprehension listening Ans:- Barriers to Listening As pointed out earlier, listening is not easy and there are a number of obstacles that stand in the way of effective listening, both within and outside the workplace. These barriers may be categorized as follows 1. Physiological Barriers This was discussed earlier under the barriers to communication. Some people may have genuine hearing problems or deficiencies that prevent them from listening properly. Once detected, they can generally be treated. Other people may have difficulty in processing information, or memory related problems which make them poor listeners. Another physiological barrier is rapid thought. Listeners have the ability to process information at the rate of approximately 500 words per minute, whereas speakers talk at around 125 words per minute. Since listeners are left with a lot of spare time, their attention may not be focused on what the speaker is saying, but may wander elsewhere. 2. Physical Barriers These refer to distractions in the environment such as the sound of an air conditioner, cigarette smoke, or an overheated room, which interfere with the listening process. They could also be in the form of information overload. For example, if you are in a meeting with your manager and the phone rings and your mobile beeps at the same time to let you know that you have a message; it is very hard to listen carefully to what is being said. 3. Attitudinal Barriers Pre-occupation with personal or work related problems can make it difficult to focus ones attention completely on what a speaker is saying, even if what is being said is of prime importance. Another common attitudinal barrier is egocentrism, or the belief that you are more knowledgeable than the speaker and that you have nothing new to learn from his ideas. People with this kind of closed minded attitude make very poor listeners. 4. Wrong Assumptions The success of communication depends on both the sender and the receiver, as we have seen in an earlier unit. It is wrong to assume that communication is the sole responsibility of the sender or the speaker and that listeners have no role to play. Such an

assumption can be a big barrier to listening. For example, a brilliant speech or presentation, however well delivered, is wasted if the receiver is not listening at the other end. Listeners have as much responsibility as speakers to make the communication successful, by paying attention, seeking clarifications and giving feedback. Another wrong assumption is to think that listening is a passive activity, in which a listener merely absorbs the thoughts of the speaker. On the contrary, real listening or active listening is hard work it requires speaking sometimes to ask questions, agree or disagree with the speaker, give feedback, etc. Yet another barrier of this type is to assume that speakers are more powerful than listeners. Speakers are seen as being in command of things, whereas listeners are seen to be weak and lacking authority. According to communication experts however, the reverse is true. Listeners are as important and as powerful as speakers. In fact David J. Schwartz, writer and management professor, emphasizes the importance of listening by saying Big people monopolize the listening. Small people monopolize the talking. 5. Cultural Barriers Accents can be barriers to listening, since they interfere with the ability to understand the meaning of words that are pronounced differently. The problem of different accents arises not only between cultures, but also within a culture. For example, in a country like India where there is enormous cultural diversity, accents may differ even between different regions and states. Another type of cultural barrier is differing cultural values. The importance attached to listening and speaking differs in western and oriental cultures. Generally, Orientals regard listening and silence as almost a virtue, whereas Westerners attach greater importance to speaking. Therefore this would interfere with the listening process, when two people from these two different cultures communicate. 6. Gender Barriers

Communication research has shown that gender can be a barrier to listening. Studies have revealed that men and women listen very differently and for different purposes. Women are more likely to listen for the emotions behind a speakers words, while men listen more for the facts and the content. Example A salesperson giving a demonstration of a new type of office equipment may be asked by two colleagues if the equipment will work without any problems and respond by saying Sure. A male user may take his answer at face value, whereas a female user may detect some hesitation in his voice. This is because the male user listens for the content of the message, whereas the female user listens for the tone of the message. 7. Lack of Training Listening is not an inborn skill. People are not born good listeners. They have to develop the art of listening through practice and training. Lack of training in listening skills is an important barrier to listening, especially in the Indian context. Lee Iacocca, former Chairman of the Chrysler Corporation in the US, was one of the first to recognize the need for organized training programs in listening skills. Today, many organizations both in India and abroad incorporate listening skills in their training programs. 8. Bad Listening Habits Most people are very average listeners who have developed poor listening habits that are hard to shed and that act as barriers to listening. For example, some people have the habit of faking attention or trying to look like a listener, in order to impress the speaker and to assure him that they are paying attention. Others may tend to listen to each and every fact and, as a result, miss out on the main point. Yet another habit is to avoid difficult listening and to tune off deliberately, if the subject is too technical or difficult to understand. Sometimes, the subject itself may be dismissed as uninteresting, because the listener does not want to listen. Differences between discriminative listening and comprehension listening

1. Discriminative Listening This is the most basic type of listening, whereby the difference between the sounds is identified. Unless the differences between the sounds are identified, the meaning expressed by such differences cannot be grasped. Once we learn to distinguish between sounds in our own language, we are able to do the same in other languages. One reason why people belonging to one country find it difficult to speak the language of another country is that they find the sounds similar and cannot understand the subtle differences. 2. Comprehension Listening Once we have learnt to discriminate between the different sounds, the next step is to try to comprehend the meaning of these sounds. In order to do this, we require a dictionary of words, along with the rules of grammar and syntax. Apart from the verbal communication, we also need to understand the meaning conveyed by the speakers nonverbal behavior. This can be achieved by closely observing various aspects of the speakers body language and tone of voice. Q5.Discuss the principles of business writing Ans:- Principles of Business Letter Writing Business letters are used primarily to communicate with external stakeholders such as consumers, intermediaries, government and bankers. The principles of business letter writing are somewhat different from the principles of writing general letters. Business letters are much more formal than general letters. Before we go into the specifics of business letter writing, let us look briefly at some of these principles* Consideration and Courtesy It is very important to retain the goodwill of customers and other external publics. A discourteous, rude letter can make you lose business. Therefore, the business letter should be extremely polite at all times and mindful of the Ps and Qs, i.e., the words please, thank you and sorry. Even if you happen to get a rude letter from a customer, you must respond politely, in order to retain the customer.

If the company has been at fault, it is important to apologize to the customer for the mistake and for the inconvenience caused. The overall tone should not be negative. For example, avoid saying We cannot grant your request. Instead state it in a more tactful way, explaining the reasons for not being able to grant the request. If you are sending a job rejection letter to a candidate, it should be worded politely and in a positive tone. Consideration means that you should appeal to the readers interest. The importance of stressing the you attitude rather than the me attitude was dealt with in an earlier unit. This is similar to the language of advertisements, which talk about the benefits of the product to the end user. For example, instead of saying We will be open 24 hours, say You can avail of round-theclock service. * Directness and Conciseness Business letters should be brief and to the point, avoiding unnecessary details and round about expressions. A typical Indian tendency is to be too wordy or verbose, using redundancies and unnecessary words. Business letters should give maximum information to the reader, using minimum words. * Clarity and Precision Business letters should be clearly worded, avoiding the use of jargon or technical terms, and slang words. Concrete words should be used, so that there is no ambiguity. Example : Instead of saying I received your communication, it is better to be more precise by saying I received your letter. The letter should include a single main idea and paragraphs should be used to elaborate on sub ideas. * Appearance Apart from the content, the format, layout and overall look of the letter should be equally appealing to the reader. Attention should be paid to the quality of paper used. The margins should be appropriate, including one inch on each side and one and a half inches on top and at the bottom. A business letter should include the following standard components

1. 2. 3.

Date in the upper right hand corner The To address above the salutation in the upper left hand corner. The Salutation When addressing a firm, Messr should be used before the name of the firm. Since business letters are formal, the appropriate salutation when addressing an individual is Dear Mr./Ms., followed by the last name, rather than the first name, which is informal. If the gender of the reader is not known, it is better to use a neutral salutation, such as Dear Customer or Investor.

4.

Sometimes, an Attention Line may be included below the salutation, in order to ensure prompt action. For example, Attention : John Smith, HR Manager.

5.

A Subject Line indicates the purpose of the letter and is placed between the salutation and the first line of the letter.

6. 7.

The Body of the letter includes an explanation of the main idea(s). The Close is the ending of the letter and should be polite and friendly, so as to retain goodwill. A standard close for a business letter is Yours faithfully or sincerely.

8.

Enclosures Sometimes, a business letter may include an enclosure such as a pamphlet or a brochure, in which case this should be indicated at the end, below the signature line, as Encl : 2, meaning two enclosures.

Q6. Explain the advantages of oral communication with the help of suitable example. Ans:- Advantages of Oral Communication To recap briefly what was discussed in Unit 2, oral communication has some advantages compared to written communication. These include its personal quality, high interactivity, possibility of making immediate contact, instantaneous feedback and control over the receivers attention. Oral communication was also classified into oral face-to-face communication (meetings and presentations) and oral non face-to-face communication (teleconferencing, telephone and voice

mail).While face-to-face meetings are more effective than non face-to-face communication in most ways, they are expensive and impractical sometimes, due to the distance factor. Thanks to advances in technology, meetings today can still take place without being face-to-face, through teleconferencing. Teleconferencing allows participants at distant locations to speak and sometimes to see each other. Apart from the high cost and the difficulty in setting it up, teleconferencing has the same advantages as oral face-to-face communication. Example Several retailers like Walmart, the worlds largest retailer, make use of teleconferencing to keep their US headquarters in touch with their store managers worldwide. Some headhunters also make use of the facility to conduct preliminary interviews and shortlist candidates based in other countries, before inviting them for a face-to-face interview. Many multi-national corporations and large Indian organizations also use this facility extensively. In spite of its advantages, teleconferencing will not replace face-to-face meetings completely, since it is unsuitable for certain types of communication that involve brainstorming, negotiations, persuasion and problem solving. Telephone communication, another form of non face-to-face communication, has the biggest advantage of being able to contact a receiver who would be impossible to reach in person. Today, mobile phones have made it even easier to contact people who are on the move. Telephone communication also has a personal quality and permits the use of some non-verbal cues such as tone of voice, to enhance the communication. Voice mail is a type of telephone communication and is similar to an answering machine. Although it is generally inferior to speaking in person to the other party, it has some advantages. When you leave a recorded message, you can make your point felt and save time that might be wasted in exchanging pleasantries. Invitations can also be declined without having to give an explanation or reason, or having the other person talk back. Thus, there is greater control over how the message is composed and delivered. Besides, voice mail also makes it possible to keep a permanent record of the communication, unlike other types of oral communication. In spite of these advantages however, voice mail has not caught on in India.

Master of Business Administration Semester I MB0039 Business Communication - 4 Credits Assignment Set - 2 (60 Marks)

Q1. List the differences between extensive reading and intensive reading Ans:- Differences between extensive reading and intensive reading Extensive Reading As we have already mentioned, our way of reading is influenced by the purpose of our reading. Most of us have the habit of reading especially when we are free, or have a lot of leisure time. We might get hold of a novel, a comic strip, or a magazine. When we read for the pure pleasure of reading, it is known as extensive reading. However, it should not be given less priority, because it is extensive reading. It is enjoyable, as well as informative. Here, we practice rapid reading to get a global/overall understanding of the matter. Intensive Reading When we read shorter texts like a research paper for getting specific details or information, we read slowly with a lot of concentration. This is known as intensive reading. When you read a book as a resource material for research, you read it intensively because the overall understanding is not the objective or purpose of our reading. When you read an article in order to write a review on it, you read it intensively. We use all the skills of reading when we do intensive reading. Q2. Explain the different advantages and disadvantages of intranet. Ans:- Advantages and Disadvantages of Intranet Like any other tool, the intranet has its advantages and limitations. Some of the primary advantages are

Discourages grapevine Grapevine, or the informal communication network was discussed in one of the earlier units. Although grapevine can be useful, it also leads to rumors, in the absence of information passed on through the formal communication network. When employees are misinformed, they become dissatisfied and de-motivated. Using the intranet as an official channel to post information for all employees to see discourages gossip and avoids creating a transparency gap.

Facilitates pre-meeting discussion The intranet may be used to discuss and debate ideas prior to a meeting, so that valuable meeting time is spent focusing only on relevant ideas. Example Suppose a sales team of five members are required to make a presentation to the Vice President Sales, on how to increase sales during the current year. Instead of each salesperson making a lengthy presentation during a meeting and then getting his ideas reviewed and approved, a discussion board can be created using the intranet, a few days before the meeting. The salespersons could post their ideas on the discussion board for the VP to review in advance, so that only the most promising ideas are discussed during the meeting. This way, the salespersons can come into the meeting more focused.

Saves time The intranet is paperless communication and is therefore a big time saver. A lot of unnecessary time wasted on filling out forms, leave requests, supply orders, etc., can be saved by doing this through the intranet. Employees can get information on official holidays and other HR related information, without having to interact with the HR manager.

Is superior to email Sending some types of information through email can sometimes create confusion and information overload. Let us take the above example of the sales team making a presentation on how to increase sales. If five salespersons each have three different ways in which they could increase sales and start emailing multiple versions of their Power Point presentations for the VP to review, it could lead to disastrous results. Instead, they could work on a shared file, by using the intranet. A central location could be created for the most recent file.

While the intranet saves valuable time and money, it is not without its down side. Before an intranet can be set up, a lot of groundwork needs to be done. You need to determine what you want it to do and how employees will use it. Some of the disadvantages are Getting started Building an intranet to your specifications can be expensive. A consultant may have to be hired to give it the desired look and feel and to make sure that it is user friendly and simple enough for all employees to use. Developing and maintaining content Once it has been set up, someone has to be responsible for maintaining and updating the information on a continuous basis. In small organizations, it may be sufficient to appoint one person to do this. In larger organizations with multiple departments, it may be necessary to appoint several persons to maintain and update information for the different departments. This becomes a complex and expensive task. Training employees Once it has been set up, employees have to be familiarized and trained on how to use the intranet. This involves investment in time and money. Convincing old economy employees Some employees may not be technology or computer savvy and may be reluctant to use the intranet to access information. Therefore, they may have to be convinced about the benefits of using the new technology. Q3. List the different principles of business letter writing.

Ans:- Principles of Business Letter Writing Business letters are used primarily to communicate with external stakeholders such as consumers, intermediaries, government and bankers. The principles of business letter writing are somewhat different from the principles of writing general letters. Business letters are much more formal than general letters. Before we go into the specifics of business letter writing, let us look briefly at some of these principles* Consideration and Courtesy It is very important to retain the goodwill of customers and other external publics. A discourteous, rude letter can make you lose business. Therefore, the

business letter should be extremely polite at all times and mindful of the Ps and Qs, i.e., the words please, thank you and sorry. Even if you happen to get a rude letter from a customer, you must respond politely, in order to retain the customer. If the company has been at fault, it is important to apologize to the customer for the mistake and for the inconvenience caused. The overall tone should not be negative. For example, avoid saying We cannot grant your request. Instead state it in a more tactful way, explaining the reasons for not being able to grant the request. If you are sending a job rejection letter to a candidate, it should be worded politely and in a positive tone. Consideration means that you should appeal to the readers interest. The importance of stressing the you attitude rather than the me attitude was dealt with in an earlier unit. This is similar to the language of advertisements, which talk about the benefits of the product to the end user. For example, instead of saying We will be open 24 hours, say You can avail of round-theclock service. * Directness and Conciseness Business letters should be brief and to the point, avoiding unnecessary details and round about expressions. A typical Indian tendency is to be too wordy or verbose, using redundancies and unnecessary words. Business letters should give maximum information to the reader, using minimum words. * Clarity and Precision Business letters should be clearly worded, avoiding the use of jargon or technical terms, and slang words. Concrete words should be used, so that there is no ambiguity. Example : Instead of saying I received your communication, it is better to be more precise by saying I received your letter. The letter should include a single main idea and paragraphs should be used to elaborate on sub ideas. * Appearance Apart from the content, the format, layout and overall look of the letter should be equally appealing to the reader. Attention should be paid to the quality of paper used. The

margins should be appropriate, including one inch on each side and one and a half inches on top and at the bottom. A business letter should include the following standard components 1. 2. 3. Date in the upper right hand corner The To address above the salutation in the upper left hand corner. The Salutation When addressing a firm, Messr should be used before the name of the firm. Since business letters are formal, the appropriate salutation when addressing an individual is Dear Mr./Ms., followed by the last name, rather than the first name, which is informal. If the gender of the reader is not known, it is better to use a neutral salutation, such as Dear Customer or Investor. 4. Sometimes, an Attention Line may be included below the salutation, in order to ensure prompt action. For example, Attention : John Smith, HR Manager. 5. A Subject Line indicates the purpose of the letter and is placed between the salutation and the first line of the letter. 6. 7. The Body of the letter includes an explanation of the main idea(s). The Close is the ending of the letter and should be polite and friendly, so as to retain goodwill. A standard close for a business letter is Yours faithfully or sincerely. 8. Enclosures Sometimes, a business letter may include an enclosure such as a pamphlet or a brochure, in which case this should be indicated at the end, below the signature line, as Encl : 2, meaning two enclosures. Q4. Write short notes on (a) corporate identity advertising (b) institutional advertising Ans:(a) Corporate Identity Advertising

This type of corporate advertising is done purely to communicate the organizations corporate identity, such as its name, logo (e.g. the Nike swoosh), trademark or brand name and slogan. When the organizations identity changes, this is communicated through corporate advertising. Example The American fast food outlet, Kentucky Fried Chicken, changed its name to KFC, to remove the association with fried, greasy chicken. This name change was communicated through corporate identity advertising. (b) Institutional Advertising This type of corporate advertising is aimed at special publics such as the media, suppliers and dealers, to correct communication problems with them. Q5. Discuss the different types of business reports Ans:- Types of Business Reports Reports may be classified based on several criteria, including their use (progress reports and financial reports), purpose (informational, analytical and persuasive reports), frequency of preparation (annual, monthly, weekly and hourly reports), length (short and long reports) and whether they are internal to the business, or are used outside the business. The most common types of business reports may be divided into the following categories 1. Periodic reports These are reports that are prepared on a regular basis, for both internal and external audiences. Their purpose is solely to inform. Examples of this type of report are a) Routine management reports These are reports such as equipment reports and sales updates and are prepared for internal audiences. b) Compliance reports These are submitted to external stakeholders, such as the government, stating compliance with regulations such as environmental norms.

c) Progress reports These reports may be prepared for both internal audiences such as top management and shareholders, as well as for external audiences such as customers. A project report stating progress on a long-term project is an example of this type of report. 2. Proposals Unlike periodic reports, the purpose of a proposal is to persuade. Proposals may be prepared for both internal and external audiences. Examples of proposals include research proposals and marketing strategy proposals to top management, proposals to the government to grant funds for building a research facility and proposals to consumers to buy a companys products. 3. Policies and Procedures The purpose of these reports is solely to inform. They are also prepared only for internal audiences. Examples include reports on company policies and procedures, written by top management and sent to all employees. This is part of downward communication. 4. Situational reports These are one-time, exceptional reports that are prepared when a unique event occurs. For example, if sales of the company has shown a significant decline, a study may be carried out to determine the reasons for declining sales and a report prepared on the findings. Similarly, a market feasibility study may be carried out before launch of a new product and a report prepared, based on the study. The purpose of such reports is usually to inform, analyze and persuade. Q6. List the different steps involved in report preparation Ans:- Steps in Report Preparation Since reports are a key to the success of any business, they should be carefully planned, organized, written and presented. A lot of groundwork should precede the actual writing of the report. We shall briefly discuss the five main steps in report preparation 1. Planning the report The first question to be asked before gathering information and writing the report, is regarding the type of report that is required. We classified reports into four main

types, based on the purpose, the audience to whom they are addressed and the frequency of the report. Secondly, it must be remembered that most reports are required by management to solve a problem, or to make a decision. Therefore, the basis, or starting point for a report is a problem. Reports are written after a problem is analyzed and a solution to the problem is found. The problem may be of a day-to-day nature, such as determining which brand of overhead projector to recommend for purchase. Or, the problem may be a negative one, such as sales of the company showing a decline. In any case, the problem is the single fundamental issue to be addressed in the report and should be clearly determined, right at the outset. Once the problem has been defined, it must be broken up into sub issues or sub problems, by asking who?. Example Suppose the purpose of a study is to survey clerical salaries in public sector banks in Bangalore city, in order to determine whether salaries in your bank are competitive and consistent. The problem may be broken up as follows What? A study of clerical salaries Why? To determine whether salaries in our firm are competitive and consistent When? Current Where? Bangalore city Who? Clerical employees in public sector banks Asking the above questions determines the exact scope of the study and reduces the problem to a workable size. the questions what, why, when, where and

The next step in planning the report is to do an audience analysis. We have seen that reports may be addressed to internal or external audiences of an organization. Some of the questions to be asked about the audience, or the reader of the report are Is the audience internal or external to the organization? Who is the specific audience or reader? - for example, top management, customers or the government? Reports written for the government and for top management should be more formal than for other audiences. Is the audience known to you? What is the level of knowledge of the audience? Is the topic familiar to the reader? If the report is of a technical nature and the reader is a layperson, the technical terms may need detailed explanation. What is the level of interest of the reader? If the report has been solicited or authorized, the readers level of interest will be high. On the other hand, if the report is voluntary or unsolicited, it may have to sustain reader interest. 2. Selecting a Method to Solve the Problem After defining the problem and doing an audience analysis, a method has to be selected to collect the necessary information to solve the problem. Broadly, information may be gathered using secondary research methods, such as books, magazines, newspapers, internet and other available sources, or through primary research methods, such as surveys that provide first hand information. 3. Gathering and Organizing Data Once the method of gathering information has been selected, the actual process of gathering the information begins. Since this is time consuming and expensive, only information that is relevant to the report and the study must be gathered. The raw data should be evaluated for its usefulness and organized in a form that is meaningful to understand. Tables, charts, graphs and summaries should be used to do this. 4. Arriving at a Conclusion Once the information has been checked for its validity and reliability, it must be interpreted and conclusions drawn. Correct interpretation of the data is needed for the success of the report. Sound conclusions cannot be made if the interpretation of

the data is faulty. A common mistake made in the interpretation of data is the tendency of the researcher to use subjective judgments, instead of objective reasoning based on facts. 5. Writing the Report The actual process of writing the report should begin only after a satisfactory solution to the problem has been found. As pointed out earlier, a well written report that contains a bad answer is worse than a badly written report that contains a good answer.

MBA SEMESTER 1 MB0040 STATISTICS FOR MANAGEMENT- 4 Credits Assignment Set- 1 (60 Marks) Q1. What are the functions of Statistics? Distinguish between Primary data and Secondary data. Ans:- Functions of Statistics Statistics is used for various purposes. It is used to simplify mass data and to make comparisons easier. It is also used to bring out trends and tendencies in the data as well as the hidden relations between variables. All this helps to make decision making much easier. Let us look at each function of Statistics in detail. 1. Statistics simplifies mass data The use of statistical concepts helps in simplification of complex data. Using statistical concepts, the managers can make decisions more easily. The statistical methods help in reducing the complexity of the data and consequently in the understanding of any huge mass of data. 2. Statistics makes comparison easier Without using statistical methods and concepts, collection of data and comparison cannot be done easily. Statistics helps us to compare data collected from different sources. Grand totals, measures of central tendency, measures of dispersion, graphs and diagrams, coefficient of correlation all provide ample scopes for comparison. 3. Statistics brings out trends and tendencies in the data After data is collected, it is easy to analyse the trend and tendencies in the data by using the various concepts of Statistics. 4. Statistics brings out the hidden relations between variables

Statistical analysis helps in drawing inferences on data. Statistical analysis brings out the hidden relations between variables. 5. Decision making power becomes easier With the proper application of Statistics and statistical software packages on the collected data, managers can take effective decisions, which can increase the profits in a business. Differences between primary and secondary data

Q2. Draw a histogram for the following distribution: Age No. people of 5 10 15 8 2 0-10 10-20 20-30 30-40 40-50

Ans;Below figure displays the histogram for the distribution of age data.

Figure: Histogram for the distribution of age We join the upper left corner of highest rectangle to the right adjacent rectangles left corner and right upper corner of highest rectangle to left adjacent rectangles right corner. From the intersecting point of these lines we draw a perpendicular to the X-axis. The X-reading at that point gives the mode of the distribution. If the widths of the rectangles are not equal then we make areas of rectangles proportional and draw the histogram Q3. Find the median value of the following set of values: 45, 32, 31, 46, 40, 28, 27, 37, 36, 41. Ans:Arranging in ascending order, we get: 27, 28, 31, 32, 36, 37, 40, 41, 45, 46 We have, n = 10

The median for the given set of values is 36.5

Q4. Calculate the standard deviation of the following data: Marks No. students of 3 15 26 23 9 4 78-80 80-82 82-84 84-86 86-88 88-90

Ans:Below table represents the frequency distribution of data required for calculating the standard deviation. Table: Frequency distribution of data ClassInterval 78-80 80-82 82-84 84-86 86-88 88-90 Midvalue X 79 81 83 85 87 89 Frequencyf 3 15 26 23 9 4 80 d = x-832 Fd fd2 -2 -1 0 1 2 3 -6 12 -15 15 0 0

23 23 18 36 12 36 32 122

Q5. An unbiased coin is tossed six times. What is the probability that the tosses will result in: (i) exactly two heads and (ii) at least five heads.

Ans:Let A be the event of getting head. Given that:

Binominal distribution is = i) The probability that the tosses will result in exactly two heads is given by:

Therefore, the probability that the tosses will result in exactly two heads is 15/64. ii) The probability that the tosses will result in at least five heads is given by:

Therefore, the probability that the tosses will result in at least five heads is 7/64. Q6. Explain briefly the types of sampling Ans;- Types of Sampling By choosing a sample technique carefully, errors can be minimised. Let us take a look at the different techniques available. The sampling techniques may be broadly classified into. i) Probability Sampling

ii) Non-Probability Sampling 1 Probability sampling Probability sampling provides a scientific technique of drawing samples from the population. The technique of drawing samples is according to the law in which each unit has a predetermined probability of being included in the sample. The different ways of assigning probability are: i) Each unit has the same chance of being selected. ii) Sampling units have varying probability iii) Units have probability proportional to the sample size We will discuss here some of the important probability sampling designs. Simple random sampling Under this technique, sample units are drawn in such a way that each and every unit in the population has an equal and independent chance of being included in the sample. If a sample unit is replaced before drawing the next unit, then it is known as Simple Random Sampling With Replacement [SRSWR]. If the sample unit is not replaced before drawing the next unit, then it is called Simple Random Sampling without replacement [SRSWOR]. In first case, probability of drawing a unit is 1/N, where N is the population size. In the second case probability of drawing a unit is 1/Nn. Stratified random sampling This sampling design is most appropriate if the population is heterogeneous with respect to characteristic under study or the population distribution is highly skewed. We subdivide the population into several groups or strata such that : i) Units within each stratum is more homogeneous

ii) Units between strata are heterogeneous iii) Strata do not overlap, in other words, every unit of population belongs to one and only one stratum The criteria used for stratification are geographical, sociological, age, sex, income and so on. The population of size N is divided into K strata relatively homogenous of size N1, N2.Nk such that N1 + N2 + + Nk = N. Then, we draw a simple random sample from each stratum either proportional to size of stratum or equal units from each stratum. The table 7.3 displays the merits and demerits of stratified random sampling. Table 7.3: Merits and demerits of stratified random sampling Merits 1. Sample is more representative Demerits 1. Many times the stratification is not effective 2. Provides more efficient estimate 2. Appropriate sample sizes are not drawn from each of the stratum 3. Administratively more convenient 4. Can be applied in situation where different degrees of accuracy is desired for different segments of population Systematic sampling This design is recommended if we have a complete list of sampling units arranged in some systematic order such as geographical, chronological or alphabetical order. Suppose the population size is N. The population units are serially numbered 1 to N in some systematic order and we wish to draw a sample of n units. Then we divide units from 1 to N into K groups such that each group has n units.

This implies nK = N or K = N/n. From the first group, we select a unit at random. Suppose the unit selected is 6th unit, thereafter we select every 6 + Kth units. If K is 20, n is 5 and N is 100 then units selected are 6, 26, 46, 66, 86. The table 7.5 displays the merits and demerits of systematic sampling. Table 7.5: Merits and demerits of systematic sampling Merits Demerits

1. Very easy to operate and easy to 1. Many case we do not get up-tocheck. 2. It saves time and labour. date list. 2. It gives biased results if periodic feature exist in the data. 3. More efficient than simple random sampling if we have up-to-date frame. Cluster sampling The total population is divided into recognisable sub-divisions, known as clusters such that within each cluster units are more heterogeneous and between clusters they are homogenous. The units are selected from each cluster by suitable sampling techniques. The figure 7.7 represents the cluster sampling where each packet of candy packet forms a cluster.

Fig. 7.7: Cluster sampling Multi-stage sampling

The total population is divided into several stages. The sampling process is carried out through several stages. It is represented as in figure 7.8.

The table 7.6 displays the merits and demerits of multi-stage sampling. Table 7.6: Merits and demerits of multi stage sampling Merits Demerits

Greater flexibility in sampling Estimates are less accurate method Existing division can be used Investigator should have knowledge of the entire population that will be sampled 2 Non-probability sampling Depending upon the object of enquiry and other considerations a predetermined number of sample units is selected purposely so that they represent the true characteristics of the population. A serious drawback of this sampling design is that it is highly subjective in nature. The selection of sample units depends entirely upon the personal convenience, biases, prejudices and beliefs of the investigator. This method will be more successful if the investigator is thoroughly skilled and experienced. Judgment Sampling The choice of sample items depends exclusively on the judgment of the investigator. The investigators experience and knowledge about the population will help to select the sample

units. It is the most suitable method if the population size is less. The table 7.7 displays the merits and demerits of judgement sampling. Table 7.7: Merits and demerits of judgement sampling Merits 1. Most useful for small population Demerits 1. It is not a scientific method.

2. Most useful to study some unknown 2. It has a risk of investigators traits of a population some of whose bias being introduced. characteristics are known. 3. Helpful in solving day-to-day

problems. Convenience sampling The sample units are selected according to convenience of the investigator. It is also called chunk which refers to the fraction of the population being investigated which is selected neither by probability nor by judgment. Moreover, a list or framework should be available for the selection of the sample. It is used to make pilot studies. However, there is a high chance of bias being introduced. Quota sampling It is a type of judgment sampling. Under this design, quotas are set up according to some specified characteristic such as age groups or income groups. From each group a specified number of units are sampled according to the quota allotted to the group. Within the group the selection of sample units depends on personal judgment. It has a risk of personal prejudice and bias entering the process. This method is often used in public opinion studies.

Master of Business Administration- MBA Semester 1 MB0040 Statistics for Management - 4 Credits Assignment Set - 2 (60 Marks)

Q1. Explain the following terms with respect to Statistics: (i) Sample, (ii) Variable, (iii) Population. Ans:- Basic Terms Used in Statistics Statistics, being a specialised subject, has a number of terms which have to be used. You need to know and understand these terms in order to do any statistical work. Let us get you acquainted with some of the basic terms used in Statistics. 1 Sample A sample is a part or subset of the population. By studying the sample, you can predict the characteristics of the entire population from where the sample is taken. The data that describes the characteristics of a sample is known as statistic. If the population is large, it is hard to collect data. Hence, a part of the population is chosen to study the characteristics of the entire population. The size of the sample can never be as large as the size of the population. Proper care must be taken while choosing the samples. In the figure 2.3, a sample of three consumers is drawn from the entire population of eight consumers. 2 Variable In a population, some characteristics remain the same for all units and some others vary from unit to unit. The quantitative characteristic that varies from unit to unit is called a variable. The qualitative characteristic that varies from unit to unit is called an attribute. A variable that assumes only some specified values in a given range is known as discrete variable. A variable that assumes all the values in the range is known as continuous variable. For example, the number of children per family and number of petals in a flower are examples of discrete variables. The height and weight of persons are examples of continuous variables.

3 Population or Universe The totality of all units or individuals in a survey is called population or universe. If the number of objects in a population is finite then it is called finite population otherwise it is known as infinite population. The data that describes the characteristics of the population is known as parameter. In the figure 2.3, the total number of eight consumers constitutes the entire population.

Fig. 2.3: Population versus sample Q2. What are the types of classification of data? Ans;- The important types of classification are: Geographical classification : Data classified according to region is geographical classification. Chronological classification : Data classified according to the time of its occurrence is called chronological classification. Conditional classification : Classification of data done according to certain conditions is called conditional classification. Qualitative classification : Classification of data that is immeasurable is called qualitative classification. For example, sex of a person, marital status, color and others.

Quantitative classification : Classification of data that is measurable either in discrete or continuous form is called quantitative classification. Statistical Series :Data is arranged logically according to size or time of occurrence or some other measurable or non-measurable characteristics. Q3. Find the (i) arithmetic mean and (ii) range of the following data: 15, 17, 22, 21, 19, 26, 20. Ans;(i) arithmetic mean Arithmetic mean is defined as the sum of all values divided by number of values and is represented by .

Before you study how to compute arithmetic mean, you have to be familiar with the terms such as discrete data, frequency and frequency distribution, which are used in this unit. If the number of values is finite, then the data is said to be discrete data. The number of occurrences of each value of the data set is called frequency of that value. A systematic presentation of the values taken by variable together with corresponding frequencies is called a frequency distribution of the variable.

(ii) range of the following data: 15, 17, 22, 21, 19, 26, 20.

The arithmetic mean

is given by:

Therefore, the arithmetic mean is 20. Q4. Suppose two houses in a thousand catch fire in a year and there are 2000 houses in a village. What is the probability that: (i) none of the houses catch fire and (ii) At least one house catch fire?

Ans;- Given the probability of a house catching fire is: and

Therefore, the required probabilities are calculated as follows: i. The probability that none catches fire is given by:

Therefore, the probability that none of the houses catches fire is 0.01832. ii. The probability that at least one catches fire is given by:

Therefore, the probability that at least one house catches fire is 0.98168.

Q5. (i) What are the characteristics of Chi-square test? Ans:- The following are the characteristics of Chi-Square test (c2 test). The c2 test is based on frequencies and not on parameters It is a non-parametric test where no parameters regarding the rigidity of population of populations are required Additive property is also found in c2 test The c2 test is useful to test the hypothesis about the independence of attributes The c2 test can be used in complex contingency tables The c2 test is very widely used for research purposes in behavioral and social sciences including business research It is defined as:

where, O is the observed frequency and E is the expected frequency. Q5 (ii) The data given in the below table shows the production in three shifts and the number of defective goods that turned out in three weeks. Test at 5% level of significance whether the weeks and shifts are independent. 1st Shift I II III Total Week 15 20 25 60 2nd Week 5 10 15 30 3rd Week 20 20 20 60 Total 40 50 60 150

Ans:- Observed and expected values required to calculate c2. Observed and expected values for data

Observed Value (O) 15 20 25 5 10 15 20 20 20

Expected Value (E)

(O E)2

40 x 60 /150 = 16 50 x 60/150 = 20 60 x 60/150 = 24 40 x 30/150 = 8 50 x 30/150 = 10 60 x 30/150 = 12 40 x 60/150 = 16 50 x 60 /150 = 20 60 x 60/150 = 24

1 0 1 9 0 9 16 0 16 c2

0.0625 0.0000 0.0417 1.1250 0.0000 0.7500 1.0000 0.0000 0.6667 3.6459

The steps followed to calculate c2 are described below. 1. Null hypothesis Ho: The week and shifts are independent Alternate hypothesis HA: The week and shifts are dependent 2. Level of Significance is 5% and D.O.F (3 1) (3 1) = 4

3. Test Statistics

4. Test c2cal = 3.6459 5. Conclusion: Since c2cal (3.6459) < c2tab (9.49), Ho is accepted. Hence, the attributes week and shifts are independent.

Q6. Find Karl Pearsons correlation co-efficient for the data given in the below table: X Y Ans:Solution: Applying the formula for r and substituting the respective values from the table we get r as: Sums related to solved problem Y 22 14 4 12 8 Y = 60 X2 400 256 144 64 16 X2 = 880 Y2 484 196 16 144 64 Y2 = 904 XY 440 224 48 96 32 XY = 840 20 22 16 14 12 4 8 12 4 8

X 20 16 12 8 4 X = 60

Solution: Applying the formula for r and substituting the respective values from the table we get r as:

Hence, Karl Pearsons correlation coefficient is 0.70.

Master of Business Administration- MBA Semester 1 MB0041 Financial and Management Accounting - 4 Credits Assignment Set- 1 (60 Marks) Q1. The Balanced Score Card is a framework for integrating measures derived from strategy. Take an Indian company which has adopted balance score card successfully and explain how it had derived benefits out of this framework. Ans:- The Balanced Score Card is a framework for integrating measures derived from strategy. While retaining financial measures of past performance, the Balanced Score Card introduces the drivers of future financial performance. (Figure 1) The drivers (customer, internal business process, learning & growth perspectives) are derived from the organizations strategy translated into objectives and measures. The Balanced Score Card is more than a measurement system it can be used as an organizing framework for their management processes. The real power of the Balanced Score Card is when it is transformed from a measurement system to a management system. It fills the void that exists in most management systems - the lack of a systematic process to implement and obtain feedback about strategy

BSC in Indian Company Tata Motors Commercial Vehicles Business Unit Consisting of three plants in India, and supported by a nationwide sales and service network, Tata Motors Commercial Vehicles Business Unit (CVBU) manufactures the full range of commercial vehicles, such as 60-seat buses and 6x4 off-road vehicles. With a workforce of over 26,000, CVBU serves over 60% of the Indian market and is one of the top 10 truck manufacturers in the world. The CVBU has a top financial objective of being among the worlds top five profitable commercial vehicle manufacturers, which is supported by growth and cost themes and objectives. Scorecard Commencement CVBU began its scorecard implementation in 2000, in support of efforts to reverse several years of poor financial performance. A new strategy, crafted by the leadership team headed by the then new Executive Director (essentially CEO) Ravi Kant focused first on turnaround to be followed by sustainable growth and profitability through being the lowest-cost producer. The scorecard was chosen as the preferred strategy implementation tool following the attendance by several senior managers at a seminar delivered by Dr. Norton.

Benefits And thus far the benefits from deploying the scorecard have been impressive. For instance, between 2001 and 2003 revenues grew by 40% (to a least double that of its nearest competitor).

Note that from April December 2002 total volume sales of commercial vehicles was 72,612 units, which rose to 104, 626 in the same timeframe in 2003 - an impressive increase of 44%. Such performance, coupled with an aggressive cost reduction programme, helped its parent, Tata Motors Corporation, turn a US$108.62 million loss into a US$65.17 million profit between 2001 and 2003. For fiscal year ended March 2005 CVBU reported a 25% increase in sales in its domestic market of 25% against industry growth of 22%. As recognition of its success in 2004 CVBU was inaugurated into the Balanced Scorecard Collaboratives prestigious Hall of Fame, one of the first two Asia-based organizations to be afforded this honour. The original Strategy Map and Balanced Scorecard was created by putting in place a high-level steering committee, comprising functional heads and some other key officers such as the regional managers in sales and marketing. The committee reported to Ravi Kant. The committee appointed a core project team of five people (from the Business Excellence Service Department and Executive Directors Office) to facilitate the scorecard creation and deployment process. Final validation and ownership would remain with Ravi Kant and the Steering Committee. At the outset the core teams knowledge was essentially drawn from the performance management and measurement expertise from the Business Excellence Service Department (the head of which sat on the Steering Committee), which had longstanding expertise in performance management frameworks such as The Malcolm Baldrige model. Indeed it was a Baldrige assessment that highlighted the business units weakness in strategy deployment. Once the CVBU Strategy Map was agreed upon, a concerted programme was conducted to provide detailed explanations to all functions and departments of the benefits of the scorecard and its importance to CVBU. Devolution

Despite widespread communication, in year one the key focus was on defining the strategic objectives and their supporting strategic initiatives, and so the scorecard remained a high-level tool. KC Girotra, who as Head of the Business Excellence Service Department leads the scorecard effort, says: It was then decided that it should be cascaded to the lowest working levels in the organization, such as the Area Offices (sales and Marketing) and Centres of Excellence/Departments (plant locations). The cascade process involved Business Excellence Service Team Members, who had been trained by external consultants, running strategy mapping workshops within plants and functions. Team members also collaborated with both the CVBU Steering Committee and managers at lower levels to put in place a review process for monitoring and analysing performance on local Balanced Scorecards. Evolution and review led, for example, in year two to a sharpening of strategic initiatives to be more in line with the strategic challenges and the introduction of comparative data as a basis for selecting targets. More than 300 scorecards have been created within CVBU covering all functions, departments, manufacturing centres of excellence and area offices (deepest work units within the sales and marketing organization). The CVBU level Balanced Scorecard defines the overall objectives, targets and timeframes to be achieved by the organization. These goals are then cascaded into Balanced Scorecards at each lower level. Hence, each scorecard is linked to the higher level scorecard. Additionally, each division/function defines its own initiatives to help achieve their own strategic objectives. For example, the CVBU level strategic objective of being a dominant player in domestic markets in all segments was cascaded into Lines of Business (LOB) scorecards where

there are specific market share targets. The performance plans prepared by individual employees and cross-functional teams align all employees and teams to the company goals. Maintaining Momentum To maintain momentum, CVBU pays close attention to the continuous communication to all employees of the scorecard approach and benefits. Each year Ravi Kant initiates the process of sharing the companys vision, mission, future directions and strategies at a town hall meeting with all employees. He repeats the communication session personally at all locations in the company to ensure that all employees have a strong, consistent, understanding of the business units longer-term and shorter-term goals. More, the scorecard core team conducts scorecard cascading workshops at each division/function, to communicate the CVBU Strategy Map, Balanced Scorecard and initiatives. Other communications systems include internal publications, intranet websites, presentations made by senior leaders, and so on. Critical Success Factors KC Girotra says that there are three critical success factors in implementing the Balanced Scorecard. The active and visible support of senior management A strong review process A knowledgeable team to drive and support scorecard deployment

As a final note, Ravi Kant is now Managing Director of the Tata Motors group. As much as anything this is an unambiguous endorsement of how successfully he led the implementation of CVBUs strategy. This is an extract from a case study that appears in the book Mastering Business in Asia: Succeeding with the Balanced Scorecard by James Creelman and Naresh Makhijani, John Wiley & Sons, 2005.

Q2. What is DuPont analysis? Explain all the ratios involved in this analysis. Your answer should be supported with the chart.

Ans:- DU PONT ANALYSIS

A method of performance measurement that was started by the DUPONT corporation in the 1920s.

With this method, assets are measured at their gross book value rather thin at net book value in order to produce a higher return on equity (ROE). It is also known as DU PONT identy.

The Du Pont analysis can be depicted via the following chart: DU PONT CHART

The apex of the Du pont chart is the teturtn on total assets (ROTA) defined as the product of the net profits margin (NPM) and total assets turnover ratio (TATR).
Net Profit Net Profit Net Sales Total Asset Net Sales Total Assets

Such decomposition helps in understanding how the return on total assets is influenced by the net profit margin and the total assets turnover ratio. A manager has basically three ways of improving operating performace in terms of ROA and ROE. These are : Increase capital asset turnover Increase operating profit margins Change financial leverage

Each of these primary drivers is impacted by the specific decisions on cost control, efficiency, productivity, marketing choices etc. Q3. Accounting Principles are the rules based on which accounting takes place and these rules are universally accepted. Explain the principles of materiality and principles of full disclosure. Explain why these two principles are contradicting each other. Your answer should be substantiated with relevant examples.

Ans:- Principles of Materiality: - While important details of financial status must be informed to all relevant parties, insignificant facts, which do not influence any decisions of the investors or any interested group, need not to b e communicated. Such less significant facts are not regarded as material facts. What is material and what is not material depends upon the nature of information and the party to whom the information is provided. While income has to be shown for income tax purposes, the amount can be rounded off to the nearest ten and fraction does not matter. The statement of account sent to a debtor contains all the details regarding invoices raised, amount outstanding during a particular period.

Principles of Full disclosure: - The business enterprise should disclose relevant information to all the parties concerned with the organization. It means that any information of substance or of interest to the average investors will have to be disclosed in the financial statements.

The company Act 1956 requires that income statement and balance sheet of a company must give a fair and true view of the state of affairs of the company. Full discloser of all relevant facts in accounts is the necessity in order to make accounting record useful. It is not a new thing , but is based on convention. Even in older times people used to speak truth and in full was incorporated in accounts too. Thus, full discloser is a very important convention.

For examples: - a hotel should report the building of a new wing, or the future acquisition of another property. A restaurant facing a lawsuit from a customer who was injured by tripping over a frayed carpet edge should disclose the contingency of the lawsuit. Similarly, is accounting practices of the current financial statements were changed and differ from those previously reported, the changes should be disclosed. Changes from one period to the next that affect current and future business operations should be reported if possible. Changes of this nature include changes made to the method used to determine depreciation expenses or to the method of inventory valuation, such changes would increase or decrease the value of ending inventory, cost of sale, gross margin and net income or loss. All changes disclosed should indicate the dollar effects such disclosures have on financial statements.

Q4. Explain any two types of errors that are disclosed by trial balance with examples and rectification entry. Note - Avoid giving examples given in the self- learning material.

Ans:- Types of Errors that are disclosed by trial balance: - Accountants prepare trial balance to checks this correctness of accounts. If total of debits balances does not agree with the total of credit balances, it is a clear cut indication that certain errors have been committed while recording the transactions the books of original entry or subsidiary books.

All errors of accounting procedure can be classified as errors of principle: When a transaction is recorded again the fundamental principles of accounting, it is an error of principle. For Example if revenue expenditure is treated as capital expenditure or vice versa. 1. Posting a wrong amount: This mistake may occur while posting an entry from subsidiary book to ledger. Example: Cash received from Krupa Rs. 1250 is posted to Krupas ledger account Rs. 1520, while its correct posted in cash a/c Rectification entry: Krupa account To Suspense a/c Dr. Rs. 270 Rs. 270

Being excess credit given to Krupa a/c rectified. 2. Omitting to post an entry from subsidiary book to ledger: If an entry made in the subsidiary book does not get posted to ledger, the trial balance does not tally. Example: Stationery bill paid Rs.2000 recorded in cash account but is not posted to Stationery account at all. Rectification entry: Stationery account To Suspense a/c Dr. Rs. 2000 Rs. 2000

Being excess credit given to Krupa a/c rectified.


Q5. Distinguish between financial accounting and management accounting

Ans:- : Distinction between Financial Accounting and Management Accounting Financial accounting is the preparation and communication of financial information to outsiders such as creditors, bankers, government, customers and so on. Another objective of financial

accounting is to give complete picture of the enterprise to shareholders. Management accounting on the other hand aims at preparing and reporting the financial data to the management on regular basis. Management is entrusted with the responsibility of taking appropriate decisions, planning, performance evaluation, control, management of costs, cost determination etc., For both financial accounting and management accounting the financial data is the same and the reports prepared in financial accounting are also used in management accounting But the following are major differences between Financial accounting and Management accounting. Financial accounting The primary users of financial accounting shareholders, government employees etc., Accounting information is Management accounting may adopt any information are Management accounting Top, middle and lower level managers use the information for planning and decision making

creditors, authorities,

always expressed in terms of money Financial data is presented for a definite period, say one year or a quarter Financial accounting focuses on historical data Financial accounting is a

measurement unit like labour hours, machine hours or product units for the purpose of analysis Reports are prepared on continuous basis, monthly or weekly or even daily

Management accounting is oriented towards future Management accounting makes use of other disciplines like economics, management,

discipline by itself and has its own principles, policies and

information system, operation research etc.,

conventions

Q6. XYZ Ltd provides the following information

Jan-01 Sundry Debtors Cash in hand Cash at Bank Bills Receivable Inventory Bills Payables Outstanding expenses Sundry Creditors Bank Overdraft Short term Loans 6,000 30,000 30,000 32,000 65,000 13,000 15,000 16,000 90,000 12,000

Dec-31 1,05,000 20,000 20,000 30,000 84,000 8,000

5,000 58,000 42,000 36,000

Prepare a schedule of changes in working capital

Hint: Net Working capital: Jan 1st 89000 and Dec31st 110000 Ans:- Schedule of changes in Working Capital

Master of Business Administration- MBA Semester 1 MB0041 Financial and Management Accounting - 4 Credits Assignment Set- 1 (60 Marks) Q1. Illustration 1: Compute the cash flow from operating activities Profit and Loss Account

To

By Sales including cash sales 5,00,000 30,000 20,000

Cost of goods sold Office expenses Selling expenses Depreciation Loss plant Goodwill off Income tax Net Profit written on sale of

4,00,000 1,00,000 12,000 8,000 6,000 Profit on sale of land Interest on investment

4,000

3,000 7,000 1,10,000 5,50,000 5,50,000

Balance Sheet as on . Mar-31 2,006 Stock Debtors Bills Receivable Creditors Bills Payable Outstanding Expenses 30,000 15,000 6,000 10,000 8,000 4,000 2007 28,000 12,000 8,000 12,000 5,000 5,000

Hint: Net cash from operating activities= 76000

Ans:-

Q2. The following extract refers to a commodity for the half year ending 31st March 2008. Prepare a cost statement. Purchase materials of raw 120,000 Direct wages Opening stock Raw materials Finished goods (1000 units) Work in progress: Closing stock: 16000 20000 100000

Rent, rate, insurance 40,000 and Works expenses

opening closing Carriage inwards Cost of factory

4,800 16000 1440 8,000

raw material F. Goods (2,000 tons) Sale of finished goods

22240

300000

Advertising, discounts allowed and selling costs Re.1 per ton sold. Production during the year is 16,000 tons. Prepare a cost sheet. Hint: Total cost or cost of sales= 255000 Profit= 45000 Sales= 300000 Ans:-

To be valued only at number of units sold. Opening stock of finished goods + production minus closing stock = Number of units sold. ** Always to be valued at number of units sold. Number of units sold x Selling price per Q3. Avon garments Ltd manufactures readymade garments and uses its cut-pieces of cloth to manufacture dolls. The following statement of cost has been prepared. Readymade Particulars garments Dolls Rs. Direct material Direct labour Variable overheads Fixed overheads Total cost Sales Profit (loss) Rs. 80,000 13,000 17,000 24,000 1,34,000 1,70,000 36,000 6,000 1,200 2,800 3,000 13,000 12,000 -1,000 Total Rs. 86,000 14,200 19,800 27,000 1,47,000 1,82,000 35,000

The cut-pieces used in dolls have a scrap value of Rs 1,000 if sold in the market. As there is a loss of Rs. 1,000 in the manufacturing of dolls, it is suggested to discontinue their manufacture. Advise the management. Hint : Total cost=Readymade garments 134000; Doll= 13000 and total=147000 Ans:- Discontinue manufacture of dolls Readymade garments Total cost Profit (loss) 134000 36000 Dolls 13000 (1000) Total 147000 35000

Q4. Describe the essential features of budgetary control. Ans:- Essential Features of Budgetary Control An effective budgeting system should have essential features to get best results. In this direction, the following may be considered as essential features of an effective budgeting. Business Policies defined: The top management of an organization strives to have an action plan for every activity and for each department. Every budget should reflect the business policies formulated from time to time. The policies should be precise and the same must be clearly defined. No ambiguity should enter the document. Clear knowledge should be provided to all the personnel concerned who are going to execute the policies. Periodic suggestions should be called for. Forecasting: Business forecasts are the foundation of budgets. Time and again discussions should be arranged to derive the most profitable combinations of forecasts. Better results can be anticipated based on the sound forecasts. As far as possible, quantitative techniques should be made use of while forecasting Formation of Budget Committee: A budget committee is a group of representatives of various important departments in an organization. The functions of committee should be specified clearly. The committee plays a vital role in the preparation and execution of budget estimated. It brings coordination among other departments. It aids in the finalization of policies and programs. Non-financial activities are also considered to make it a wholesome affair. Accounting System: To make the budget a successful document, there should be proper flow of accurate and timely information. The accounting adopted by the organization should be proper and must be fine-tuned from time to time Organizational efficiency: To make the budget preparation and its subsequent implementation a success, an efficient, adequate and best organization is necessary a budgeting system should always be supported by a sound organizational structure. There must be a clear cut demarcation

of lines of authority and responsibility. There must also be a delegation of authority from top to bottom line. Management Philosophy: Every management should set a healthy philosophy while opting for the budget. Management must wholeheartedly support the activities which developing a budget. Encouragement should flow from top management. All the members must be involved to make it a workable preposition and a dream-driven document. Reporting system: Proper feedback system should be established. Provision should be made for corrective measures whenever comparative measures are proposed. Availability of statistical information: Since budgets are always prepared and expressed in quantitative terms, it is essential that sufficient and accurate relevant data should be made available to each department. Motivation: Since budget acts as a mirror, the entire organization should become smart in its approach. Every employees both executive and non-executives should be made part of the overall exercise. Employees should be persuaded than pressurized to appreciate the benefits of the budgets so that the fruits can be shared by all the members of the organization. Q5. Briefly describe labor mix variance and yield variance. Ans:- Labour Mix Variance This variance arises only when different types of workers (women and men workers, trained, semi-trained and untrained workers, are employed in manufacturing. If actual working force of different grades of workers is not in the pre-determined ratio, then the mix variance will occur. The variance shows to the management as to how much of the labor cost variance is due to the changes in the composition of labor force. It is calculated as follows: LMV = (Revised standard hours actual hours worked) x standard hourly rate Shorten (RSLH ALH) x SR

Where revised standard hour = total time of actual worker / total time of standard workers x standard labor rate. Labour Yield Variance This is due to the difference in the standard output specified and the actual output obtained. The formula is as follows: LYV = (Actual output Standard output) x standard cost per unit

Q6. How is standard costing related to budgetary control? Ans:- Both are closely interrelated. They both aim at the improvement of the system of managerial control. They both achieve the same objective of maximum efficiency and cost control by establishing pre-determined standards. They compare actual performance with the predetermined standard. They take necessary steps to improve the situation wherever necessary. Both techniques are forward looking. However, the following are some of the differences identified. 1. The scope of budgetary control is wider. It is integrated plan of action, a coordinated plan in respect of all functions of an enterprise. The scope of standard costing on the other hand is limited to the operating level. Here too, it is further linked to costs. Budgetary control is extensive whereas standard costing is intensive in its application 2. Budgetary control deals with costs and revenues. But standard costing restricts only with costs. 3. Budgetary control takes into account all activities such as production, sales, purchases, finance, capital expenditure, personnel whereas standard costing is restricted to deal with only costs. 4. Budgetary control targets are based on past actual adjusted to future trends. In standard costing, standards are based on technical assessment.

5. At the approach level, budgeted targets work as the maximum limit of expenses above which the actual expenditure should not normally exceed. Under standard costing, standards are attainable level of performance. 6. Budget is projection of final accounts. Standard costs are projection of only cost accounts. 7. Budgetary control emphasizes the forecasting aspect of the future operations. Standard 8. Costing scope and utility is limited to only operating level of the concern. 9. In budgetary control, the degree of variance analysis tends to be much less and variances are not revealed through the accounts but are revealed in total. But in standard costing, variances are analyzed in details according to their originating causes and are revealed through different accounts. 10. Budgetary control is possible even in parts of expenses according to the attitude of management. A standard costing system can not be operated in parts. All items of expenditure included in cost units are to be accounted for.

Master of Business Administration - Semester 1 MB 0042: Managerial Economics ASSIGNMENT- Set 1 Q 1. Define Managerial Economics and explain its main characteristics. Ans:- Managerial economics is a science that deals with the application of various economic theories, principles, concepts and techniques to business management in order to solve business and management problems. It deals with the practical application of economic theory and methodology to decision-making problems faced by private, public and non-profit making organizations. The same idea has been expressed by Spencer and Seigelman in the following words. Managerial Economics is the integration of economic theory with business practice for the purpose of facilitating decision making and forward planning by the management. According to Mc Nair and Meriam, Managerial economics is the use of economic modes of thought to analyze business situation. Brighman and Pappas define managerial economics as, the application of economic theory and methodology to business administration practice. Joel dean is of the opinion that use of economic analysis in formulating business and management policies is known as managerial economics. Features of managerial Economics 1. It is more realistic, pragmatic and highlights on practical application of various economic theories to solve business and management problems. 2. It is a science of decision-making. It concentrates on decision-making process, decisionmodels and decision variables and their relationships. 3. It is both conceptual and metrical and it helps the decision-maker by providing measurement of various economic variables and their interrelationships. 4. It uses various macro economic concepts like national income, inflation, deflation, trade cycles etc to understand and adjust its policies to the environment in which the firm operates.

5. It also gives importance to the study of non-economic variables having implications of economic performance of the firm. For example, impact of technology, environmental forces, socio-political and cultural factors etc. 6. It uses the services of many other sister sciences like mathematics, statistics, engineering, accounting, operation research and psychology etc to find solutions to business and management problems. It should be clearly remembered that Managerial Economics does not provide ready-made solutions to all kinds of problems faced by a firm. It provides only the logic and methodology to find out answers and not the answers themselves. It all depends on the managers ability, experience, expertise and intelligence to use different tools of economic analysis to find out the correct answers to business problems. Q 2. State and explain the law of demand. Ans:- The Law of Demand It explains the relationship between price and quantity demanded of a commodity. It says that demand varies inversely with the price. The law can be explained in the following manner: Keeping other factors that affect demand constant, a fall in price of a product leads to increase in quantity demanded and a rise in price leads to decrease in quantity demanded for the product. The law can be expressed in mathematical terms as Demand is a decreasing function of price. Symbolically, thus D = F (p) where, D represents Demand, P stands for Price and F denotes the Functional relationship. The law explains the cause and effect relationship between the independent variable [price] and the dependent variable [demand]. The law explains only the general tendency of consumers while buying a product. A consumer would buy more when price falls due to the following reasons: 1. A product becomes cheaper.[Price effect] 2. Purchasing power of a consumer would go up.[Income effect] 3. Consumers can save some amount of money.

4. Cheaper products are substituted for costly products [substitution effect]. Important Features of Law of Demand 1. There is an inverse relationship between price and quantity demanded. 2. Price is an independent variable and demand is a dependent variable 3. It is only a qualitative statement and as such it does not indicate quantitative changes in price and demand. 4. Generally, the demand curve slopes downwards from left to right. The operation of the law is conditioned by the phrase Other things being equal. It indicates that given certain conditions, certain results would follow. The inverse relationship between price and demand would be valid only when tastes and preferences, customs and habits of consumers, prices of related goods, and income of consumers would remain constant. Exceptions to the Law of Demand Generally speaking, customers would buy more when price falls in accordance with the law of demand. Exceptions to law of demand states that with a fall in price, demand also falls and with a rise in price demand also rises. This can be represented by rising demand curve. In other words, the demand curve slopes upwards from left to right. It is known as an exceptional demand curve or unusual demand curve. It is clear from the diagram that as price rises from Rs. 4.00 to Rs. 5.00, quantity demanded also expands from 10 units to 20 units.

1. Giffens Paradox A paradox is an inconsistency or contrary. Sir Robert Giffen, an Irish Economist, with the help of his own example (inferior goods) disproved the law of demand. The Giffens paradox holds that Demand is strengthened with a rise in price or weakened with a fall in price. He gave the example of poor people of Ireland who were using potatoes and meat as daily food articles. When price of potatoes declined, customers instead of buying larger quantities of potatoes started buying more of meat (superior goods). Thus, the demand for potatoes declined in spite of fall in its price. 2. Veblens effect Thorstein Veblen, a noted American Economist contends that there are certain commodities which are purchased by rich people not for their direct satisfaction, but for their snob appeal or ostentation.Veblens effect states that demand for status symbol goods would go up with a rise in price and vice-versa. In case of such status symbol commodities it is not the price which is important but the prestige conferred by that commodity on a person makes him to go for it. More commonly cited examples of such goods are diamonds and precious stones, world famous paintings, commodities used by world famous personalities etc. Therefore, commodities having snob appeal are to be considered as exceptions to the law of demand. 3. Fear of shortage When serious shortages are anticipated by the people, (e.g., during the war period) they purchase more goods at present even though the current price is higher. 4. Fear of future rise in price

If people expect future hike in prices, they buy more even though they feel that current prices are higher. Otherwise, they have to pay a still high price for the same product. 5. Speculation Speculation implies purchase or sale of an asset with the hope that its price may rise or fall and make speculative profit. Normally speculation is witnessed in the stock exchange market. People buy more shares only when their prices show a rising trend. This is because they get more profit, if they sell their shares when the prices actually rise. Thus, speculation becomes an exception to the law of demand. 6. Conspicuous consumption Conspicuous consumption are those items which are purchased by consumers even though their prices are rising on account of their special uses in our modern style of life. In case of articles like wrist watches, scooters, motorcycles, tape recorders, mobile phones etc, customers buy more in spite of their high prices. 7. Emergencies During emergency periods like war, famine, floods, cyclone, accidents etc., people buy certain articles even though the prices are quite high. 8. Ignorance Sometimes people may not be aware of the prices prevailing in the market. Hence, they buy more at higher prices because of sheer ignorance. 9. Necessaries Necessaries are those items which are purchased by consumers what ever may be the price. Consumers would buy more necessaries in spite of their higher prices. Changes or Shifts in Demand

It is to be clearly understood that if demand changes only because of changes in the price of the given commodity, in that case there would be either expansion or contraction in demand. Both of them can be explained with the help of only one demand curve. If demand changes not because of price changes but because of other factors or forces, then in that case there would be either increase or decrease in demand. If demand increases, there would be forward shift in the demand curve to the right and if demand decreases, then there would be backward shift in the demand cure.

Q 3. What is Demand Forecasting? Explain in brief various methods of forecasting demand. Ans:- Demand forecasting seeks to investigate and measure the forces that determine sales for existing and new products. Generally companies plan their business - production or sales in anticipation of future demand. Hence forecasting future demand becomes important. The art of successful business lies in avoiding or minimizing the risks involved as far as possible and face the uncertainties in a most befitting manner. Thus Demand Forecasting refers to an estimation of most likely future demand for product under given conditions. Methods or Techniques of Forecasting Demand forecasting is a highly complicated process as it deals with the estimation of future demand. It requires the assistance and opinion of experts in the field of sales management. While estimating future demand, one should not give too much of importance to either statistical information, past data or experience, intelligence and judgment of the experts. Demand

forecasting, to become more realistic should consider the two aspects in a balanced manner. Application of commonsense is needed to follow a pragmatic approach in demand forecasting. Broadly speaking, there are two methods of demand forecasting. They are: 1. Survey methods and 2. Statistical methods

1. Survey Methods Survey methods help us in obtaining information about the future purchase plans of potential buyers through collecting the opinions of experts or by interviewing the consumers. These methods are extensively used in short run and estimating the demand for new products. There are different approaches under survey methods. They are Consumers interview method:

Under this method, efforts are made to collect the relevant information directly from the consumers with regard to their future purchase plans. In order to gather information from consumers, a number of alternative techniques are developed from time to time. Among them, the following are some of the important ones. a) Survey of buyers intentions or preferences: It is one of the oldest methods of demand forecasting. It is also called Opinion surveys. Under this method, consumer-buyers are requested to indicate their preferences and willingness about particular products. They are asked to reveal their future purchase plans with respect to specific items. They are expected to give answers to questions like what items they intend to buy, in what quantity, why, where, when, what quality they expect, how much money they are planning to spend etc. Generally, the field survey is conducted by the marketing research department of the company or hiring the services of outside research organizations consisting of learned and highly qualified professionals. The heart of the survey is questionnaire. It is a comprehensive one covering almost all questions either directly or indirectly in a most intelligent manner. It is prepared by an expert body who are specialists in the field or marketing. The questionnaire is distributed among the consumer buyers either through mail or in person by the company. Consumers are requested to furnish all relevant and correct information. The next step is to collect the questionnaire from the consumers for the purpose of evaluation. The materials collected will be classified, edited analyzed. If any bias prejudices, exaggerations, artificial or excess demand creation etc., are found at the time of answering they would be eliminated. The information so collected will now be consolidated and reviewed by the top executives with lot of experience. It will be examined thoroughly. Inferences are drawn and conclusions are arrived at. Finally a report is prepared and submitted to management for taking final decisions. The success of the survey method depends on many factors:

1) The nature of the questions asked, 2) The ability of the surveyed 3) The representative of the samples 4) Nature of the product 5) characteristics of the market 6) consumer-buyers behavior, their intentions, attitudes, thoughts, motives, honesty etc. 7) Techniques of analysis conclusions drawn etc.

The management should not entirely depend on the results of survey reports to project future demand. Consumer buyers may not express their honest and real views and as such they may give only the broad trends in the market. In order to arrive at right conclusions, field surveys should be regularly checked and supervised. This method is simple and useful to the producers who produce goods in bulk. Here the burden of forecasting is put on customers. However this method is not much useful in estimating the future demand of the households as they run in large numbers and also do not freely express their future demand requirements. It is expensive and also difficult. Preparation of a questionnaire is not an easy task. At best it can be used for short term forecasting. b) Direct Interview Method Experience has shown that many customers do not respond to questionnaire addressed to them even if it is simple due to varied reasons. Hence, an alternative method is developed. Under this method, customers are directly contacted and interviewed. Direct and simple questions are asked to them. They are requested to answer specifically about their budget, expenditure plans, particular items to be selected, the quality and quantity of products, relative price preferences etc.

for a particular period of time. There are two different methods of direct personal interviews. They are as follows: i) Complete enumeration method Under this method, all potential customers are interviewed in a particular city or a region. The answers elicited are consolidated and carefully studied to obtain the most probable demand for a product. The management can safely project the future demand for its products. This method is free from all types of prejudices. The result mainly depends on the nature of questions asked and answers received from the customers. However, this method cannot be used successfully by all sellers in all cases. This method can be employed to only those products whose customers are concentrated in a small region or locality. In case consumers are widely dispersed, this method may not be physically adopted or prove costly both in terms of time and money. Hence, this method is highly cumbersome in nature. ii) Sample survey method or the consumer panel method Experience of the experts show that it is impossible to approach all customers; as such careful sampling of representative customers is essential. Hence, another variant of complete enumeration method has been developed, which is popularly known as sample survey method. Under this method, different cross sections of customers that make up the bulk of the market are carefully chosen. Only such consumers selected from the relevant market through some sampling method are interviewed or surveyed. In other words, a group of consumers are chosen and queried about their preferences in concrete situations. The selection of a few customers is known as sampling. The selected consumers form a panel. This method uses either random sampling or the stratified sampling technique. The method of survey may be direct interview or mailed questionnaire to the selected consumers. On the basis of the views expressed by these selected consumers, most likely demand may be estimated. The advantage of a panel lies in the fact that the same panel is continued and new expensive panel does not have to be formulated every time a new product is investigated.

As compared to the complete enumeration method, the sample survey method is less tedious, less expensive, much simpler and less time consuming. This method is generally used to estimate short run demand by government departments and business firms. Success of this method depends upon the sincere co-operation of the selected customers. Hence, selection of suitable consumers for the specific purpose is of great importance. Even with careful selection of customers and the truthful information about their buying intention, the results of the survey can only be of limited use. A sudden change in price, inconsistency in buying intentions of consumers, number of sensible questions asked and dropouts from the panel for various reasons put a serious limitation on the practical usefulness of the panel method. Collective opinion method or opinion survey method This is a variant of the survey method. This method is also known as Sales force polling or Opinion poll method. Under this method, sales representatives, professional experts and the market consultants and others are asked to express their considered opinions about the volume of sales expected in the future. The logic and reasoning behind the method is that these salesmen and other people connected with the sales department are directly involved in the marketing and selling of the products in different regions. Salesmen, being very close to the customers, will be in a position to know and feel the customers reactions towards the product. They can study the pulse of the people and identify the specific views of the customers. These people are quite capable of estimating the likely demand for the products with the help of their intimate and friendly contact with the customers and their personal judgments based on the past experience. Thus, they provide approximate, if not accurate estimates. Then, the views of all salesmen are aggregated to get the overall probable demand for a product. Further, these opinions or estimates collected from the various experts are considered, consolidated and reviewed by the top executives to eliminate the bias or optimism and pessimism of different salesmen. These revised estimates are further examined in the light of factors like proposed change in selling prices, product designs and advertisement programs, expected changes in the degree of competition, income distribution, population etc. The final sales forecast

would emerge after these factors have been taken into account. This method heavily depends on the collective wisdom of salesmen, departmental heads and the top executives. It is simple, less expensive and useful for short run forecasting particularly in case of new products. The main drawback is that it is subjective and depends on the intelligence and awareness of the salesmen. It cannot be relied upon for long term business planning. Delphi Method or Experts Opinion Method This method was originally developed at Rand Corporation in the late 1940s by Olaf Helmer, Dalkey and Gordon. This method was used to predict future technological changes. It has proved more useful and popular in forecasting non-economic rather than economic variables. It is a variant of opinion poll and survey method of demand forecasting. Under this method, outside experts are appointed. They are supplied with all kinds of information and statistical data. The management requests the experts to express their considered opinions and views about the expected future sales of the company. Their views are generally regarded as most objective ones. Their views generally avoid or reduce the Halo Effects and Ego Involvement of the views of the others. Since experts opinions are more valuable, a firm will give lot of importance to them and prepare their future plan on the basis of the forecasts made by the experts. End Use or Input Output Method Under this method, the sale of the product under consideration is projected on the basis of demand surveys of the industries using the given product as an intermediate product. The demand for the final product is the end use demand of the intermediate product used in the production of the final product. An intermediate product may have many end users, For e.g., steel can be used for making various types of agricultural and industrial machinery, for construction, for transportation etc. It may have the demand both in the domestic market as well as international market. Thus, end use demand estimation of an intermediate product may

involve many final goods industries using this product, at home and abroad. Once we know the demand for final consumption goods including their exports we can estimate the demand for the product which is used as intermediate good in the production of these final goods with the help of input output coefficients. The input output table containing input output coefficients for particular periods are made available in every country either by the Government or by research organizations. This method is used to forecast the demand for intermediate products only. It is quite useful for industries which are largely producers goods, like aluminum, steel etc. The main limitation of the method is that as the number of end users of a product increase, it becomes more inconvenient to use this method. 2. Statistical Method It is the second most popular method of demand forecasting. It is the best available technique and most commonly used method in recent years. Under this method, statistical, mathematical models, equations etc are extensively used in order to estimate future demand of a particular product. They are used for estimating long term demand. They are highly complex and complicated in nature. Some of them require considerable mathematical background and competence. They use historical data in estimating future demand. The analysis of the past demand serves as the basis for present trends and both of them become the basis for calculating the future demand of a commodity in question after taking into account of likely changes in the future. There are several statistical methods and their application should be done by some one who is reasonably well versed in the methods of statistical analysis and in the interpretation of the results of such analysis. Trend Projection Method An old firm operating in the market for a long period will have the accumulated previous data on either production or sales pertaining to different years. If we arrange them in chronological order,

we get what is called time series. It is an ordered sequence of events over a period of time pertaining to certain variables. It shows a series of values of a dependent variable say, sales as it changes from one point of time to another. In short, a time series is a set of observations taken at specified time, generally at equal intervals. It depicts the historical pattern under normal conditions. This method is not based on any particular theory as to what causes the variables to change but merely assumes that whatever forces contributed to change in the recent past will continue to have the same effect. On the basis of time series, it is possible to project the future sales of a company. Further, the statistics and information with regard to the sales call for further analysis. When we represent the time series in the form of a graph, we get a curve, the sales curve. It shows the trend in sales at different periods of time. Also, it indicates fluctuations and turning points in demand. If the turning points are few and their intervals are also widely spread, they yield acceptable results. Here the time series show a persistent tendency to move in the same direction. Frequency in turning points indicates uncertain demand conditions and in this case, the trend projection breaks down. The major task of a firm while estimating the future demand lies in the prediction of turning points in the business rather than in the projection of trends. When turning points occur more frequently, the firm has to make radical changes in its basic policy with respect to future demand. It is for this reason that the experts give importance to identification of turning points while projecting the future demand for a product. The heart of this method lies in the use of time series. Changes in time series arise on account of the following reasons: 1. Secular or long run movements: Secular movements indicate the general conditions and direction in which graph of a time series move in relatively a long period of time. 2. Seasonal movements: Time series also undergo changes during seasonal sales of a company. During festival season, sales clearance season etc., we come across most unexpected changes.

3. Cyclical Movements: It implies change in time series or fluctuations in the demand for a product during different phases of a business cycle like depression, revival, boom etc. 4. Random movement: When changes take place at random, we call them irregular or random movements. These movements imply sporadic changes in time series occurring due to unforeseen events such as floods, strikes, elections, earth quakes, droughts and such other natural calamities. Such changes take place only in the short run. Still they have their own impact on the sales of a company. An important question in this connection is how to ascertain the trend in time series? A statistician, in order to find out the pattern of change in time series may make use of the following methods. 1. The Least Squares method. 2. The Free hand method. 3. The moving average method. 4. The method of semi averages. The method of Least Squares is more scientific, popular and thus more commonly used when compared to the other methods. It uses the straight line equation Y= a + bx to fit the trend to the data. Economic Indicators Economic indicators as a method of demand forecasting are developed recently. Under this method, a few economic indicators become the basis for forecasting the sales of a company. An economic indicator indicates change in the magnitude of an economic variable. It gives the signal about the direction of change in an economic variable. This helps in decision making process of a company. We can mention a few economic indicators in this context. 1. Construction contracts sanctioned for demand towards building materials like cement.

2. Personal income towards demand for consumer goods. 3. Agriculture income towards the demand for agricultural inputs, instruments, fertilizers, manure, etc. 4. Automobile registration towards demand for car spare parts, petrol etc. 5. Personal Income, Consumer Price Index, Money supply etc., towards demand for consumption goods. The above mentioned and other types of economic indicators are published by specialist organizations like the Central Statistical Organization. The analyst should establish relationship between the sale of the product and the economic indicators to project the correct sales and to measure as to what extent these indicators affect the sales. The job of establishing relationship is a highly difficult task. This is particularly so in case of new products where there are no past records. Under this method, demand forecasting involves the following steps: a. The forecaster has to ensure whether a relationship exists between the demand for a product and certain specified economic indicators. b. The forecaster has to establish the relationship through the method of least square and derive the regression equation. Assuming the relationship to be linear, the equation will be y = a + bx. c. Once the regression equation is obtained by forecasting the value of x, economic indicator can be applied to forecast the values of Y. i.e. demand. d. Past relationship between different factors may not be repeated. Therefore, the value judgment is required to forecast the value of future demand. In addition to it, many other new factors may also have to be taken into consideration.

When economic indicators are used to forecast the demand, a firm should know whether the forecasting is undertaken for a short period or long period. It should collect adequate and appropriate data and select the ideal method of demand forecasting. The next stage is to determine the most likely relationship between the dependent variables and finally interpret the results of the forecasting. However it is difficult to find out an appropriate economic indicator. This method is not useful in forecasting demand for new products. Q4. Define the term equilibrium. Explain the changes in market equilibrium and effects of shifts in supply and demand. Ans:- Meaning of equilibrium The word equilibrium is derived from the Latin word aequilibrium which means equal balance. It means a state of even balance in which opposing forces or tendencies neutralize each other. It is a position of rest characterized by absence of change. It is a state where there is complete agreement of the economic plans of the various market participants so that no one has a tendency to revise or alter his decision. In the words of professor Mehta: Equilibrium denotes in economics absence of change in movement. Changes in Market Equilibrium The changes in equilibrium price will occur when there will be shift either in demand curve or in supply curve or both: Effects of Shift in demand Demand changes when there is a change in the determinants of demand like the income, tastes, prices of substitutes and complements, size of the population etc. If demand raises due to a change in any one of these conditions the demand curve shifts upward to the right. If, on the other hand, demand falls, the demand curve shifts downward to the left. Such rise and fall in demand are referred to as increase and decrease in demand.

A change in the market equilibrium caused by the shifts in demand can be explained with the help of a diagram.

Effects of Changes in Both Demand and Supply Changes can occur in both demand and supply conditions. The effects of such changes on the market equilibrium depend on the rate of change in the two variables. If the rate of change in demand is matched with the rate of change in supply there will be no change in the market equilibrium, the new equilibrium shows expanded market with increased quantity of both supply and demand at the same price. This is made clear from the diagram below:

Similar will be the effects when the decrease in demand is greater than the decrease in supply on the market equilibrium.

Q 5. Explain features of LAC curve with a diagram.

Ans:-

Features of long run AC curves:

1. Tangent curve: Different SAC curves represent different operational capacities of different plants in the short run. LAC curve is locus of all these points of tangency. The SAC curve can never cut a LAC curve though they are tangential to each other. This implies that for any given level of output, no SAC curve can ever be below the LAC curve. Hence, SAC cannot be lower than the LAC in the long run. Thus, LAC curve is tangential to various SAC curves.

2. Envelope curve: It is known as Envelope curve because it envelopes a group of SAC curves appropriate to different levels of output.

3. Flatter U-shaped or dish-shaped curve: The LAC curve is also U shaped or dish shaped cost curve. But It is less pronounced and much flatter in nature. LAC gradually falls and rises due to economies and diseconomies of scale.

4. Planning curve: The LAC cure is described as the Planning Curve of the firm because it represents the least cost of producing each possible level of output. This helps in producing optimum level of output at the minimum LAC. This is possible when the entrepreneur is selecting the optimum scale plant. Optimum scale plant is that size where the minimum point of SAC is tangent to the minimum point of LAC.

5. Minimum point of LAC curve should be always lower than the minimum point of SAC curve: This is because LAC can never be higher than SAC or SAC can never be lower than LAC. The LAC curve will touch the optimum plant SAC curve at its minimum point.

A rational entrepreneur would select the optimum scale plant. Optimum scale plant is that size at which SAC is tangent to LAC, such that both the curves have the minimum point of tangency. In

the diagram, OM2 is regarded as the optimum scale of output, as it has the least per unit cost. At OM2 output LAC = SAC. LAC curve will be tangent to SAC curves lying to the left of the optimum scale or right side of the optimum scale. But at these points of tangency, neither LAC is minimum nor will SAC be minimum. SAC curves are either rising or falling indicating a higher cost Managerial Use of LAC: The study of LAC is of greater importance in managerial decision making process. 1. It helps the management in the determination of the best size of the plant to be constructed or when a new one is introduced in getting the minimum cost output for a given plant. But it is interested in producing a given output at the minimum cost. 2. The LAC curve helps a firm to decide the size of the plant to be adopted for producing the given output. For outputs less than cost lowering combination at the optimum scale i.e., when the firm is working subject to increasing returns to scale, it is more economical to under use a slightly large plant operating at less than its minimum cost output than to over use smaller unit. Conversely, at output beyond the optimum level, that is when the firm experience decreasing return to scale, it is more economical to over use a slightly smaller plant than to under use a slightly larger one. Thus, it explains why it is more economical to over use a slightly small plant rather than to under use a large plant. 3. LAC is used to show how a firm determines the optimum size of the plant. An optimum size of plant is one that helps in best utilization of resources in the most economical manner. Long Run Marginal cost:

A long-run marginal cost curve can be derived from the long-run average cost curve. Just as the SMC is related to the SAC, similarly the LMC is related to the LAC and, therefore, we can derive the LMC directly from the LAC. In the diagram we have taken three plant sizes (for the sake of simplicity) and the corresponding three SAC and SMC curves. The LAC curve is drawn by enveloping the family of SAC curves. The points of tangency between the SAC and the LAC curves indicate different outputs for different plant sizes. If the firm wants to produce ON output in the long run, it will have to choose the plant size corresponding to SAC1. The LAC curve is tangent to SAC1 at point A. For ON output, the average cost is NA and the corresponding marginal cost is NB If LAC curve is tangent to SAC1 curve at point A, the corresponding LMC curve will have to be equal to SMC1 curve at point B. The LMC will pass through point B. In other words, where LAC is equal to SAC curve (for a given output) the LMC will have to be equal to a given SMC. If output OQ is to be produced in the long run, it will be done at point c which is the point of tangency between SAC2 and the LAC. At point C, the short run average cost (SAC2) and the short-run marginal cost (SMC2) are equal and, therefore, the LAC for output OQ is QC and the corresponding LMC is also QC. The LMC curve will, therefore pass through point C. Finally, for output OR,at point D the LAC is tangent to SAC3. For OR output at point E LMC is passing through SMC3. By connecting points B ,C and E, we can draw the long-run marginal cost curve.

Cost of Production: Formulas

Q 6. Explain cost output relationship with reference to a. Total fixed cost and output b. Total variable cost and output c. .Total cost and output

Ans:A. Total fixed cost and output: TFC refers to total money expenses incurred on fixed inputs like plant, machinery, tools & equipments in the short run. Total fixed cost corresponds to the fixed inputs in the short run production function. TFC remains the same at all levels of output in the short run. It is the same when output is nil. It indicates that whatever may be the quantity of output, whether 1 to 6 units, TFC remains constant. The TFC curve is horizontal and parallel to OX-axis, showing that it is constant regardless of output per unit of time. TFC starts from a point on Y-axis indicating that the total fixed cost will be incurred even if the output is zero. In our example, Rs 360=00 is TFC. It is obtained by summing up the product or quantities of the fixed factors multiplied by their respective unit price.

B. Total variable cost and output: TVC refers to total money expenses incurred on the variable factor inputs like raw materials, power, fuel, water, transport and communication etc, in the short run. Total variable cost corresponds to variable inputs in the short run production function. It is obtained by summing up the production of quantities of variable inputs multiplied by their prices. The formula to calculate TVC is as follows. TVC = TC-TFC. TVC = f (Q) i.e. TVC is an increasing function of output. In other words TVC varies with output. It is nil, if there is no production. Thus, it is a direct cost of output. TVC rises sharply in the beginning, gradually in the middle and sharply at the end in accordance with the law of variable proportion. The law of variable proportion explains that in the beginning to obtain a given quantity of output, relative variation in variable factors-needed are in less proportion, but after a point when the diminishing returns operate, variable factors are to be employed in a larger proportion to increase the same level of output. TVC curve slope upwards from left to right. TVC curve rises as output is expanded. When output is Zero, TVC also will be zero. Hence, the TVC curve starts from the origin.

C. Total cost and output: The total cost refers to the aggregate money expenditure incurred by a firm to produce a given quantity of output. The total cost is measured in relation to the production function by multiplying the factor prices with their quantities. TC = f (Q) which means that the T.C. varies with the output. Theoretically speaking TC includes all kinds of money costs, both explicit and implicit cost. Normal profit is included in the total cost as it is an implicit cost. It includes fixed as well as variable costs. Hence, TC = TFC +TVC.

TC varies in the same proportion as TVC. In other words, a variation in TC is the result of variation in TVC since TFC is always constant in the short run.

The total cost curve is rising upwards from left to right. In our example the TC curve starts from Rs. 360-00 because even if there is no output, TFC is a positive amount. TC and TVC have same shape because an increase in output increases them both by the same amount since TFC is constant. TC curve is derived by adding up vertically the TVC and TFC curves. The vertical distance between TVC curve and TC curve is equal to TFC and is constant throughout because TFC is constant.

Master of Business Administration - Semester 1 MB 0042: Managerial Economics ASSIGNMENT- Set 2 Q 1. Explain the relationship between revenue concepts and price elasticity of demand. Ans:- Relationship between Revenue Concepts and Price Elasticity of Demand Elasticity of Demand, Average Revenue and Marginal Revenue There is a very useful relationship between elasticity of demand, average revenue and marginal revenue at any level of output. Elasticity of demand at any point on a consumers demand curve is the same thing as the elasticity on the given point on the firms average revenue curve. With the help of the point elasticity of demand, we can study the relationship between average revenue, marginal revenue and elasticity of demand at any level of output.

In the diagram AR and MR respectively are the average revenue and the marginal revenue curves. Elasticity of demand at point R on the average revenue curve = RT/Rt Now in the triangles PtR and MRT. tPR = RMT (right angles) tRP = RTM (corresponding angles) PtR= MRT (being the third angle) Therefore, triangles PtR and MRT are equiangular.

Hence RT / Rt = RM / tP In the triangles PtK and KRQ PK = RK PKt = RKQ (vertically opposite) tPK = KRQ (right angles ) Therefore, triangles PtK and RQK are congruent (i.e., equal in all respects). Hence Pt = RQ Elasticity at R = RT / Rt = RM / tP = RM / RQ

It is clear from the diagram that Hence elasticity at R = RM / RM QM It is also clear from the diagram that RM is average revenue and QM is the marginal revenue at the output OM which corresponds to the point R on the average revenue curve. Therefore elasticity at R = Average Revenue / Average Revenue Marginal Revenue If A stands for Average Revenue, M stands for Marginal Revenue and e stands for point elasticity on the average revenue curve Then e = A / A M. Thus, elasticity of demand is equal to AR over AR minus MR. By using the above elasticity formula, we can derive the formula for AR and MR separately.

e=

This can be changed into (through cross multiplication)

eA eM = A bringing As together, we have

eA A = eM A ( e 1 ) = eM A = eM / e 1 A =M (e / e 1) Therefore Average Revenue or price = M (e / e 1) Thus the price (i.e., AR) per unit is equal to marginal revenue x elasticity over elasticity minus one. The marginal revenue formula can be written straight away as M = A ((e 1) / e) The general rule therefore is: at any output, Average Revenue = Marginal Revenue x (e / e 1) and Marginal Revenue = Average Revenue x (e 1 / e) Where, e stands for point elasticity of demand on the average revenue curve. With the help of these formulae, we can find marginal revenue at any point from average revenue at the same point, provided we know the point elasticity of demand on the average revenue curve. Suppose that the price of a product is Rs.8 and the elasticity is 4 at that price. Marginal revenue will be: M = A (( e 1) / e) = 8 (( 4 1 / 4) = 8 x 3 /4 = 24 / 4 = 6. Marginal Revenue is Rs. 6.

Suppose that the price of a product is Rs.4 and the elasticity coefficient is 2 then the corresponding MR will be: M = A ( ( e-1) / e) = 4 ( ( 2 1) / 4) =4x1/4 =4/4 = 1 Marginal revenue is Rs. 1 Suppose that the price of commodity is Rs.10 and the elasticity coefficient at that price is 1 MR will be: M = A ( ( e-1) / e) =10 ( (1-1) /1) =10 x 0/1 =0 Whenever elasticity of demand is unity, marginal revenue will be zero, whatever be the price(or AR). It follows from this that if a demand curve shows unitary elasticity throughout its length the corresponding marginal revenue will be zero throughout, that is, the x axis itself will be the marginal revenue curve. Thus, the higher the elasticity coefficient, the closer is the MR to AR / price. When elasticity coefficient is one for any given price, the corresponding marginal revenue will be zero, marginal revenue is always positive when the elasticity coefficient is greater than one and marginal revenue is always negative when the elasticity coefficient is less than one. Kinked Demand curve and the corresponding Marginal Revenue curve

We measure quantity on the x axis and price on the Y axis. The demand curve AD has a kink at point B, thus exhibiting two different characteristics. From A to B it is elastic but from B to D it is inelastic. Because the demand is elastic from A to B a very small fall in price causes a very big rise in demand, but to realize the same increase in demand a very big fall in price is required as the demand curve assumes inelastic shape after point B. The corresponding marginal revenue curve initially falls smoothly, though at a greater rate. In the diagram there is a gap in MR between output 300 and 350. Generally an Oligopolist who faces a kinked demand curve will make a good gain when he reduces the price a little before the kink (point B), but if he lowers the price below B; the rival firms will lower their prices too; accordingly the price cutting firm will not be able to increase its sales correspondingly or may not be able to increase its sales at all. As a result, the demand curve of price cutting firm below B is more inelastic. The corresponding MR curve is not smooth but has a gap or discontinuity between G and L. In certain cases, the kinked demand curve may show a high elasticity in the lower portion of the demand curve beyond the kink and low elasticity in higher portion of the demand curve before the kink Marginal revenue to such a demand curve will show a gap but instead of at a lower level, it will start at a higher level.

Relationship between AR, MR, TR and Elasticity of Demand In the diagram AR is the average revenue curve, MR is the marginal revenue curve and OD is the total revenue curve. At the middle point C of average revenue curve elasticity is equal to one. On its lower half it is less than one and on the upper half it is greater than one. MR corresponding to the middle point C of the AR curve is zero. This is shown by the fact that MR curve cuts the x axis at Q which corresponds to the point C on the AR curve. If the quantity is greater than OQ it will correspond to that portion of the AR curve where e<1 marginal revenue is negative because MR goes below the x axis. Likewise for a quantity less than OQ, e>1 and the marginal revenue is positive. This means that if quantity greater than OQ is sold, the total revenue will be diminishing and for a quantity less than OQ the total revenue TR will be increasing. Thus the total revenue TR will be maximum at the point H where elasticity is equal to one and marginal revenue is zero. Significance of Revenue curves The relationship between price elasticity of demand and total revenue is important because every firm has to decide whether to increase or decrease the price depending on the price elasticity of demand of the product. If the price elasticity of demand for his product is relatively elastic it will be advantageous to reduce price as it increases his total revenue. On the other hand, if the price elasticity of demand for his product is relatively inelastic he should raise the price as it increases his total revenue.

Average revenue, which is the price per unit, considered along with average cost will show to the firm whether it is profitable to produce and sell. If average revenue is greater than average cost, the firm is getting excess profit; if it is less than average cost, the firm is running at a loss. Firms profit is maximum at a point where Marginal revenue is equal to Marginal cost. Any increase in output beyond that point will mean loss on additional units produced; restriction of output before that point will mean lower profit. Thus the concept of average revenue is relevant to find out whether the firm is running on profit or loss; the concept of marginal revenue together with marginal cost will show profit maximizing output for the firm. Q 2. Explain the emergence of Consumers surplus with their practical application. Ans:- It is a parallel concept to consumers surplus. Producers surplus is owners surplus. It may be defined as the excess or surplus income received by a seller over above the price at which he is willing to sell a product. It is the different between the actual price at which he is selling and the price at which he is willing to sell Hence, it arises when the actual price received exceeds the minimum price that the seller is ready to accept. Emergence of consumers surplus and producers surplus A simple example may be given to show the emergence of both these surpluses Vivek is a seller and is ready to sell his CD at Rs. 20-00 Vivek now sells the CD at Rs. 30-00. Producers = 10-00 Vinay is ready to pay Rs. 40-00 for the same CD Vinay now buy the CD at Rs. 30-00 Consumers surplus = Price ready to pay price actually paid. surplus = price received price ready to charge.30-00 20-00

= 40-00 30-00 = 10-00.

The concepts of both consumers surplus and producers surplus can be explained with the help of demand and supply curves simultaneously.

In the above diagram, price is represented on Y Axis and quantity demanded and supplied on X Axis. Demand and supply are equal at E. OR is the equilibrium price and OQ is the equilibrium quantity demanded and supplied. In the diagram. The consumer is ready to pay OD price and he actually pays OR price. Hence, consumers surplus is RDE ie, area A. Generally consumers surplus is the area under demand curve and above the market price line. Hence, the area A is the consumers surplus in the diagram. The seller is ready to accept the minimum price of OS and sells it at OR price. Hence, producers surplus is SRE, ie, the area B. Producers surplus is the area above the supply curve and below the market price line Hence, the area B is producers surplus. Thus, with the help of one diagram we can explain the emergence of both surpluses. Q 3. What is Monetary policy? Explain the general objectives of monetary policy. Ans:- Monetary policy is a part over all economic policy of a country. It is employed by the government as an effective tool to promote economic stability and achieve certain predetermined objectives. General objectives of monetary policy. 1. Neutral money policy: Prof. Wicksteed, Hayak, Robertson and others have advocated this policy. This objective was in vogue during the days of gold standard. According to this policy, money is only a technical devise having no other role to play. It should be a passive factor

having only one function, namely to facilitate exchange. It should not inject any disturbances. It should be neutral in its effects on prices, income, output, and employment. They considered that changes in total money supply are the root cause for all kinds of economic fluctuations and as such if money supply is stabilized and money becomes neutral, the price level will vary inversely with the productive power of the economy. If productivity increases, cost per unit of output declines and prices fall and vice-versa. According to this policy, money supply is not rigidly fixed. It will change whenever there are changes in productivity, population, improvements in technology etc to neutralize fundamental changes in the economy. Under these conditions, increase or decrease in money supply is allowed to result in either fall or raise in general price level. In a dynamic economy, this policy cannot be continued and it is highly impracticable in the present day economy. 2. Price stability: With the suspension of the gold standard, maintenance of domestic price level has become an important aim of monetary policy all over the world. The bitter experience of 1920s and 1930s has made all most all economies to go for price stability. Both inflation and deflation are dangerous and detrimental to smooth economic growth. They distort and disturb the working of the economic system and create chaos. Both of them are bad as they bring unnecessary loss to some groups where as undue advantage to some others. They have potential power to create economic inequality, political upheavals and social unrest in any economy. In view of this, price stability is considered as one of the main objectives of monetary policy in recent years. It is to be remembered that price stability does not mean that prices of all commodities are kept constant or fixed over a period of time. It refers to the absence of sharp variations or fluctuations in the average price level in the country. A hundred percent price stability is neither possible nor desirable in any economy. It simply implies relative price stability. A policy of price stability checks cyclical fluctuations and smoothen production and distribution, keeps the value of money stable, prevent artificial scarcity or prosperity, makes economic calculations possible, introduces an element of certainty, eliminate socioeconomic disturbances, ensure equitable distribution of income and wealth, secure social justice and promote economic welfare. On account of all these benefits, monetary authorities have to take concrete steps to check price oscillations. Price stability is considered as one of the prerequisite condition for economic development and it contributes positively to the attainment

of a steady rate of growth in an economy. This is because price stability will build up public morale and instill confidence in the minds of people, boost up business activity, expand various kinds of economic activities and ensure distributive justice in the country. Prof Basu rightly observes, A monetary policy which can maintain a reasonable degree of price stability and keep employment reasonably full, sets the stage of economic development. 3. Exchange rate stability: Maintenance of stable or fixed exchange rate was one of the major objects of monetary policy for a long time under the gold standard. The stability of national output and internal price level was considered secondary and subservient to the former. It was through free and automatic imports and exports of gold that the country was able to remove the disequilibrium in the balance of payments and ensure stability of exchange rates with other countries. The government followed the policy of expanding currency and credit with the inflow of gold and contracting currency and credit with the outflow of gold. In view of suspension of gold standard and IMF mechanism, this object has lost its significance. However, in order to have smooth and unhindered international trade and free flow of foreign capital in to a country, it becomes imperative for a county to maintain exchange rate stability. Changes in domestic prices would affect exchange rates and as such there is great need for stabilizing both internal price level and exchange rates. Frequent changes in exchange rates would adversely affect imports, exports, inflow of foreign capital etc. Hence, it should be controlled properly. 4. Control of trade cycles: Operation of trade cycles has become very common in modern economies. A very high degree of fluctuations in over all economic activities is detrimental to the smooth growth of any economy. Economic instability in the form of inflation, deflation or stagflation etc would serve as great obstacles to the normal functioning of an economy. Basically, changes in total supply of money are the root cause for business cycles and its dampening effects on the entire economy. Hence, it has become one of the major objectives of monetary authorities to control the operation of trade cycles and ensure economic stability by regulating total money supply effectively. During the period of inflation, a policy of contraction in money supply and during the period of deflation, a policy of expansion in money supply has to be adopted. This would create the necessary economic stability for rapid economic development.

5. Full employment: In recent years it has become another major goal of monetary policy all over the world especially with the publication of general theory by Lord Keynes. Many wellknown economists like Crowther, Halm. Gardner Ackley, William, Beveridge and Lord Keynes have strongly advocated this objective in the context of present day situations in most of the countries. Advanced countries normally work at near full employment conditions. Their major problem is to maintain this high level of employment situation through various economic polices. This object has become much more important and crucial in developing countries as there is unemployment and under employment of most of the resources. Deliberate efforts are to be made by the monetary authorities to ensure adequate supply of financial resources to exploit and utilize resources in the best possible manner so as to raise the level of aggregate effective demand in the economy. It should also help to maintain balance between aggregate savings and aggregate investments. This would ensure optimum utilization of all kinds of resources, higher national output, income and higher living standards to the common man. 6. Equilibrium in the balance of payments: This objective has assumed greater importance in the context of expanding international trade and globalization. To day most of the countries of the world are experiencing adverse balance of payments on account of various reasons. It is a situation where in the import payments are in excess of export earnings. Most of the countries which have embarked on the road to economic development cannot do away with imports on a large scale. Imports of several items have become indispensable and without these imports their development process will be halted. Hence, monetary authorities have to take appropriate monetary measures like deflation, exchange depreciation, devaluation, exchange control, current account and capital account convertibility, regulate credit facilities and interest rate structures and exchange rates etc. In order to achieve a higher rate of economic growth, balance of payments equilibrium is very much required and as such monetary authorities have to take suitable action in this direction. 7. Rapid economic growth: This is comparatively a recent objective of monetary policy. Achieving a higher rate of per capita output and income over a long period of time has become one of the supreme goals of monetary policy in recent years. A higher rate of economic growth would ensure full employment condition, higher output, income and better living standards to the people. Consequently, monetary authorities have to take the necessary steps to raise the

productive capacity of the economy, increase the level of effective demand for various kinds of goods and services and ensure balance between demand for and supply of goods and services in the economy. Also they should take measures to increase the rate of savings, capital formation, step up the volume of investment, direct credit money into desired directions, regulate interest rate structure, minimize economic and business fluctuations by balancing demand for money and supply of money, ensure price and overall economic stability, better and full utilization of resources, remove imperfections in money and capital markets, maintain exchange rate stability, allow the inflow of foreign capital into the country, maintain the growth of money supply in consistent with the rate of growth of output minimize adversity in balance of payments condition, etc. Depending upon the conditions of the economy money supply has to be changed from time to time. A flexible policy of monetary expansion or contraction has to be adopted to meet a particular situation. Thus, a growth-friendly monetary policy has to be pursued by monetary authorities in order to stimulate economic growth. It is to be noted that the above-mentioned objectives are inter related, inter dependent and inter connected with each other. Each one of the objectives would affect the other and in its turn is influenced by the others. Many objectives would come in clash with others under certain circumstances. A proper balance between different objectives becomes imperative. Monetary authorities have to determine the priorities depending upon the economic environment in a country. Thus, there is great need for compromise between different objectives Q 4. What is a business cycle ? Describe the different phases of a business cycle. Ans:- The term business cycle refers to a wave like fluctuation in the over all level of economic activity particularly in national output, income, employment and prices that occur in a more or less regular time sequence. It is nothing but rhythmic fluctuations in the aggregate level of economic activity of a nation. Different writers have defined business cycles in different ways. According to Prof. Haberler, The business cycle in the general sense may be defined as an alternation of periods of prosperity and depression of good and bad trade. In the words of Prof. Gordon, Business cycles consists of recurring alternations of expansion and contraction in aggregate economic activity, the alternating movements in each direction being self- reinforcing and pervading virtually all parts of the economy. According to Keynes A

trade cycle is composed of periods of good trade characterized by rising prices and low unemployment percentages, alternating with periods of bad trade characterized by falling prices and high unemployment percentages. Thus, one can notice a common feature in all these definitions, i.e., variations in the aggregate level of economic activities in different magnitudes Phases of Trade Cycle Basically, a business cycle has only two parts- expansion and contraction or prosperity and depression. Burns and Mitchell observe that peaks and troughs are the two main mark-off points of a business cycle. The expansion phase starts from revival and includes prosperity and boom. Contraction phase includes recession, depression and trough. In between these two main parts, we come across a few other interrelated transitional phases. In its broader perspective, a business cycle has five phases. They are as follows.

1. Depression, contraction or downswing It is the first phase of a trade cycle. It is a protracted period in which business activity is far below the normal level and is extremely low. According to Prof. Haberler depression is a state of affairs in which the real income consumed or volume of production per head and the

rate of employment are falling and are sub-normal in the sense that there are idle resources and unused capacity, especially unused labor. This period is characterized by: a. A sharp reduction in the volume of output, trade and other transactions. b. An increase in the level of unemployment. c. A sharp reduction in the aggregate income of the community especially wages and profits. In a few cases, profits turns out to be negative. d. A drop in prices of most of the products and fall in interest rates. e. A steep decline in consumption expenditure and fall in the level of aggregative effective demand. f. A decline in marginal efficiency of capital and hence the volume of investment. g. Absence of incentives for production as the market has become dull. h. A low demand for Loanable funds, surplus cash balances with banks leading to a contraction in the creation of bank credit. i. A high rate of business failures. j. An increasing difficulty in returning old debts by the debtors. This forces them to sell their inventories in the market where prices are already falling. This deepens depression further. k. A decline in the level of investment in stocks as it becomes less attractive and less profitable. This reduces the deposits with the banks and other financial institutions leading to a contraction in bank credit. l. A lot of excess capacity exists in capital and consumer goods industries which work much below their capacity due to lack of demand.

During depression, all construction activities come to a more or less halting stage. Capital goods industries suffer more than consumer goods industries. Since costs are sticky and do not fall as rapidly as prices, the producers suffer heavy losses. Prices of agricultural goods fall rapidly than industrial goods. During this period purchasing power of money is very high but the general purchasing power of the community is very low. Thus, the aggregate level of economic activity reaches its rock bottom position. It is the stage of trough. The economy enters the phase of depression, as the process of depression is complete. It is also called, the period of slump. During this period, there is disorder, demoralization, dislocation and disturbances in the normal working of the economic system. Consequently, one can notice all-round pessimism, frustration and despair. The entire atmosphere is gloomy and hopes are less. It is a period of great suffering and hardship to the people. Thus, it is the worst and most fearful phase of the business cycle. USA experienced depression two times, between 1873- 1879 and 1929 1933. 2. Recovery or revival Depression cannot last long, forever. After a period of depression, recovery starts. It is a period where in, economic activities receive stimulus and recover from the shocks. This is the lower turning point from depression to revival towards upswing. Depression carries with itself the seeds of its own recovery. After sometime, the rays of hope appear on the business horizon. Pessimism is slowly replaced by optimism. Recovery helps to restore the confidence of the business people and create a favorable climate for business ventures. The recovery may be initiated by the following factors: a. Increase in government expenditure so as to increase purchasing power in the hands of consumers. b. Changes in production techniques and business strategies. c. Diversification in investments or Investment in new regions. d. Explorations and exploitation of new sources of energy etc.

e. New innovations- developing new products or services, new marketing strategy etc. As a result of these factors, business people take more risks and invest more. Low wages and low interest rates, low production costs, recovery in marginal efficiency of capital etc induce the business people to take up new ventures. In the early phase of the revival, there is considerable excess capacity in the economy so, the output increases without a proportionate increase in total costs. Repairs, renewals and replacement of plants take place. Increase in government expenditure stimulates the demand for consumption goods, which in its turn pushes up the demand for capital goods. Construction activity receives an impetus. As a result, the level of output, income, employment, wages, prices, profits, start rising. Rise in dividends induce the producers to float fresh investment proposals in the stock market. Recovery in stock market begins. Share prices go up. Optimistic expectations generate a favorable climate for new investment. Attracted by the profits, banks lend more money leading to a high level of investment. The upward trends in business give a sort of fillip to economic activity. Through multiplier and acceleration effects, the economy moves upward rapidly. It is to be noted that revival may be slow or fast, weak or strong; the wave of recovery once initiated begins to feed upon itself. Generally, the process of recovery once started takes the economy to the peak of prosperity. 3. Prosperity or Full-employment The recovery once started gathers momentum. The cumulative process of recovery continues till the economy reaches full employment. Full employment may be defined as a situation where in all available resources are fully employed at the current wage rate. Hence, achieving full employment has become the most important objective of all most all economies. Now, there is all round stability in output, wages, prices, income, etc. According to Prof. Haberler Prosperity is a state of affair in which the real income consumed, produced and the level of employment are high or rising and there are no idle resources or unemployed workers or very few of either. During the period of prosperity an economy experiencesa. A high level of output, income, employment and trade. b. A high level of purchasing power, consumption expenditure and effective demand.

c. A high level of Marginal Efficiency of Capital and volume of investment. d. A period of mild inflation sets in leading to a feeling of optimism among businessmen and industrialists. e. An increase in the level of inventories of both inputs and outputs. f. A rise in Interest Rate. g. A large expansion in bank credit and financial institutions lend more money to business men. h. Firms operate almost at full capacity along with its production possibility frontier. i. Share markets give handsome gains to investors as dividends and share prices go up. Consequently, idle funds find their way to productive investments. j. A state of exuberance and enthusiasm exists in business community. k. Industrial and commercial activity, both speculative and non-speculative show remarkable expansion. l. There is all round expansion, development, growth and prosperity in the economy. Every one seems to be happy during this period. The USA experienced the longest period of prosperity between 1923 &29. 4. Boom or Over full Employment or Inflation The prosperity phase does not stop at full employment. It gives way to the emergence of a boom. It is a phase where in there will be an artificial and temporary prosperity in an economy. Business optimism stimulates further investment leading to rapid expansion in all spheres of business activities during the stage of full employment, unutilized capacity gradually disappears. Idle resources are fully employed. Hence, rise in investment can only mean increased pressure for the available men and materials. Factor inputs become scarce commanding higher

remuneration. This leads to a rise in wages and prices. Production costs go up. Consequently, higher output is obtained only at a higher cost of production. Once full employment is reached, a further increase in the demand for factor inputs will lead to an increase in prices rather than an increase in output and income. Demand for Loanable funds increases leading to a rise in interest rates. Now there will be hectic economic activity. Soon a situation develops in which the number of jobs exceed the number of workers available in the market. Such a situation is known as overfull employment or hyper-employment. During this phase: a. Prices, wages, interest, incomes, profits etc. move in the upward direction. b. MEC raises leading to business expansion. c. Business people borrow more and invest. This adds fuel to the fire. The tempo of boom reaches new heights. d. There is higher output, income and employment. Living standards of the people also increases. e. There is higher purchasing power and the level of effective demand will reach new heights. f. There is an atmosphere of over optimism all round, which results in over investment. Cost of living increases at a rate relatively higher than the increase in household incomes. e. It is a symptom of the end of prosperity phase and the beginning of recession. The boom carries with it the gems of its own destruction. The prosperity phase comes to an end when the forces favoring expansion becomes progressively weak. Bottlenecks begin to appear. Scarcity of factor inputs and rise in their prices disturb the cost calculations of the entrepreneurs. Now the entrepreneurs realize that they have over stepped the mark and become over cautious and their over-optimism paves the way for their pessimism. Thus, prosperity digs its own grave. Generally the failure of a company or a bank bursts the boom and ushers in a recession. USA experienced prosperity between 1923 and 1929.

5. Recession A turn from prosperity to Depression The period of recession begins when the phase of prosperity ends. It is a period of time where in the aggregate level of economic activity starts declining. There is contraction or slowing down of business activities. After reaching the peak point, demand for goods decline. Over investment and production creates imbalance between supply and demand. Inventories of finished goods pile up. Future investment plans are given up. Orders placed for new equipments and raw materials and other inputs are cancelled. Replacement of worn out capital is postponed. The cancellation of orders for the inputs by the producers of consumer goods creates a chain reaction in the input market. Incomes of the factor inputs decline this creates demand recession. In order to get rid of their high inventories, and to clear off their bank obligations, producers reduce market prices. In anticipation of further fall in prices, consumers postpone their purchases. Production schedules by firms are curtailed and workers are laid-off. Banks curtail credit. Share prices decline and there will be slackness in stock and financial market. Consequently, there will be a decline in investment, employment, income and consumption. Liquidity preference suddenly develops. Multiplier and accelerator work in the reverse direction. Unemployment sets in the capital goods industries and with the passage of time, it spreads to other industries also. The process of recession is complete. The wave of pessimism gets transmitted to other sectors of the economy. The whole economic system thereby runs in to a crisis. Failure of some business creates panic among businessmen and their confidence is shaken. Business pessimism during this period is characterized by a feeling of hesitation, nervousness, doubt and fear. Prof. M. W. Lee remarks, A recession, once started, tends to build upon itself much as forest fire. Once under way, it tends to create its own drafts and find internal impetus to its destructive ability. Once the recession starts, it becomes almost difficult to stop the rot. It goes on gathering momentum and finally converts itself in to a full- fledged depression, which is the period of utmost suffering for businessmen. Thus, now we have a full description about a business cycle. The USA experienced one of the severe recessions during 1957-58.

Lord Over stone describes the course of business cycle in the following words A state of quiescence (inert or silent) next improvement growing confidence prosperity excitement overtrading convulsion pressure distress ending again in quiescence A detailed study of the various phases of a business cycle is of paramount importance to the management. It helps the management to formulate various anti-cyclical measures to be taken up to check the adverse effects of a trade cycle and create the necessary conditions for ensuring stability in business. Q 5. What is inflation? Explain the causes of inflation. Ans:- Inflation has become a global phenomenon in recent years. Inflation is a sin; every government denounces it and every government practices it. Prof. ML. Stigum. Development economics is very much associated with inflation. An in-depth study of inflation is of paramount importance to a student of managerial economics. The term inflation is used in many senses and hence it is very difficult to give generally accepted, universally agreeable and precise definition to the term inflation. Popularly inflation is associated with high prices, which causes a decline in the value of money. Inflation is commonly understood as a situation of substantial and rapid general increase in the level of prices and consequent deterioration in the value of money over a period of time. It refers to the average rise in the general level of prices and fall in the value of money. Prof.Crowther defines inflation as a state in which the value of money is falling i.e. Prices are rising. Prof. Samuelson puts it thus, inflation occurs when the general level of prices and the cost is rising. According to Prof. Parkin and Bade, Inflation is an upward movement in the average level of prices. Its opposite is deflation, a downward movement in the average level of prices. Thus, the common feature of inflation is rise in prices and the degree of inflation may be measured by price indices. Causes of Inflation I Demand side

Increase in aggregative effective demand is responsible for inflation. In this case, aggregate demand exceeds aggregate supply of goods and services. Demand rises much faster than the supply. We can enumerate the following reasons for increase in effective demand. 1. Increase in money supply: Supply of money in circulation increases on account of the following reasons deficit financing by the government, expansion in public expenditure, expansion in bank credit and repayment of past debt by the government to the people, increase in legal tender money and public borrowing. 2. Increase in disposable income: Aggregate effective demand rises when disposable income of the people increases. Disposable income rises on account of the following reasons reduction in the rates of taxes, increase in national income while tax level remains constant and decline in the level of savings. 3. Increase in private consumption expenditure and investment expenditure: An increase in private expenditure both on consumption and on investment leads to emergence of excess demand in an economy. When business is prosperous, business expectations are optimistic and prices are rising, more investment is made by private entrepreneurs causing an increase in factor prices. When the incomes of the factors rise, there is more expenditure on consumer goods. 4. Increase in Exports: An increase in the foreign demand for a countrys exports reduces the stock of goods available for home consumption. This creates shortages in the country leading to rise in price level. 5. Existence of Black Money: The existence of black money in a country due to corruption, tax evasion, black-marketing etc, increases the aggregate demand. People spend such unaccounted money extravagantly thereby creating un-necessary demand for goods and services causing inflation. 6. Increase in Foreign Exchange Reserves: It may increase on account of the inflow of foreign money in to the country. Foreign Direct Investment may increase and non-resident deposits may also increase due to the policy of the government.

7. Increase in population growth creates increase in demand for every thing in a country. 8. High rates of indirect taxes would lead to rise in prices. 9. Reduction in the rates of direct taxes would leave more cash in the hands of people inducing them to buy more goods and services leading to an increase in prices. 10. Reduction in the level of savings creates more demand for goods and services. II. Supply side Generally, the supply of goods and services do not keep pace with the ever-increasing demand for goods and services. Thus, supply does not match with the demand. Supply falls short of demand. Increase in supply of goods and services may be limited because of the following reasons. 1. Shortage in the supply of factors of production When there is shortage in the supply of factors of production like raw materials, labor, capital equipments etc. there will be a rise in their prices. Thus, when supply falls short of demand, a situation of excess demand emerges creating inflationary pressures in an economy. 2. Operation of law of diminishing returns When the law of diminishing returns operate, increase in production is possible only at a higher cost which de motivates the producers to invest in large amounts. Thus production will not increase proportionately to meet the increase in demand. Hence, supply falls short of demand. 3. Hoardings by Traders and speculators During the period of shortage and rise in prices, hoarding of essential commodities by traders and speculators with the object of earning extra profits in future creates artificial scarcity of commodities. This creates a situation of excess demand paving the way for further inflation. 4. Hoarding by Consumers

Consumers may also hoard essential goods to avoid payment of higher prices in future. This leads to increase in current demand, which in turn stimulate prices. 5. Role of Trade unions Trade union activities leading to industrial unrest in the form of strikes and lockouts also reduce production. This will lead to creation of excess demand that eventually brings a rise in the price level. 6. Role of natural Calamities Natural calamities such as earthquake, floods and drought conditions also affect adversely the supplies of agricultural products and create shortage of food grains and raw materials, which in turn creates inflationary conditions. 7. War: During the period of war, shortage of essential goods create rise in prices. 8. International factors also would cause either shortage of goods and services or rise in the prices of factor inputs leading to inflation. E.g., High prices of imports. 9. Increase in prices of inputs with in the country. III. Role of Expectations Expectations also play a significant role in accentuating inflation. The following points are worth mentioning: 1. If people expect further rise in price, the current aggregate demand increases which in its turn causes a raise in the prices. 2. Expectations about higher wages and salaries affect very much the prices of related goods. 3. Expectations of wage increase often induce some business houses to increase prices even before upward wage revisions are actually made.

Thus, many factors are responsible for escalation of prices. Q 6. Write short notes on the following: a. Monopoly b. Oligopoly

Ans:a. Monopoly The word monopoly is made up of two syllables MONO means single and POLY means to sell. Thus, monopoly means existence of a single seller in the market. Monopoly is that market form in which a single producer controls the whole supply of a single commodity which has no close substitutes. Monopoly may be defined as a condition of production in which a person or a number of persons acting in combination have the power to fix the price of the commodity or the output of the commodity. It is a situation where there exists a single control over the market producing a commodity having no substitutes and no possibilities for any one to enter the industry to compete. According to Prof. Watson A monopolist is the only producer of a product that has no close substitutes. Features of Monopoly 1. Anti-Thesis of competition : Absence of competition in the market creates a situation of monopoly and hence the seller faces no threat of competition. 2. Existence of a single seller : There will be only one seller in the market who exercises single control over the market. 3. Absence of substitutes : There are no close substitutes for his product with a strong cross elasticity of demand. Hence, buyers have no alternatives.

4. Control over supply : He will have complete control over output and supply of the commodity. 5. Price Maker :The monopolist is the price maker and in taking decisions on price fixation, he is independent. He can set the price to the best of his advantage. Hence, he can either charge a high price for all customers or adopt price discrimination policy. 6. Entry barriers :Entry of other firms is barred somehow. Hence, monopolist will not have direct competitors or direct rivals in the market. 7. Firm and industry is same :There will be no difference between firm and an industry. 8. Nature of firm : The monopoly firm may be a proprietary concern, partnership concern, Joint Stock Company or a public utility which pursues an independent price-output policy. 9. Existence of super normal profits There will be place for supernormal profits under monopoly, because market price is greater than cost of production. There are different kinds of monopolies Private and public, pure monopoly, simple monopoly and discriminatory monopoly. It is to be clearly understood that with the exception of public utilities or institutions of a similar nature, whose price is set by regulatory bodies, monopolies rarely exist. Just like perfect competition, pure monopoly does not exist. Hence, we make a detailed study of simple monopoly and discriminatory monopoly in the foregoing analysis. b. Oligopoly The term oligopoly is derived from two Greek words Oligoi means a few and Poly means to sell. Under oligopoly, we come across a few producers specializing in the production of identical goods or differentiated goods competing with one another. The products traded by the oligopolists may be differentiated or homogeneous. In the case of former, we can give the e.g., of automobile industry where different model of cars, ambassador, fiat etc., are manufactured. Other examples are cigarettes, refrigerators, T.V. sets etc., pure or homogeneous

oligopoly includes such industries as cooking and commercial gas cement, food, vegetable oils, cable wires, dry batteries, petroleum etc., In the modern industrial set up there is a strong tendency towards oligopoly market situation. To avoid the wastes of competition in case of competitive industries and to face the emergence of new substitutes in case of monopoly industries, oligopoly market is developed. e.g., an electric refrigerator, automatic washing machines, radios etc. Characteristics of Oligopoly 1. Interdependence: Each and every firm has to be conscious of the reactions of its rivals. Since the number of firms is very few, any change in price, output, product etc., by one firm will have direct effect on the policy of other firms. Therefore, economic calculations must be made always with reference to the reactions of the rival firms, as they have a high degree of cross elasticitys of demand for their products. 2. Indeterminateness of the demand curve: Under oligopoly, there will be the element of uncertainty. Firms will not be knowing the particular factors which could affect demand. Naturally rise or fall in the demand for the product cannot be speculated. Changes that would be taking place may be contrary to the expected changes in the product curve.. Thus, the demand curve for the product will be indeterminate or indefinite. Prof. Sweezy explains it as a kinky demand curve. 3. Conflicting attitude of firms: Under oligopoly, on the one hand, firms may realize the disadvantages of competition and rivalry and desire to unite together to maximize their profits. On the other hand firms guided by individualistic considerations may continuously come in clash and conflict with one another. This creates uncertainty in the market. 4. Element of monopoly and competition:Under oligopoly, a firm has some monopoly power over the product it produces but not on the entire market. But monopoly power enjoyed by the firm will be limited by the extent of competition. 5. Price rigidity: Generally, prices tend to be sticky or rigid under oligopoly. This is because of the fact that if one firm changes its price, other firms may also resort to the same technique.

6. Aggressive or defensive marketing methods: Firms resort to aggressive and sometimes defensive marketing methods in order to either increase their share of the market or to prevent a decline of their share in the market. If one adopts extensive advertisement and sales promotion policy it provokes others to do the same. Prof. Boumal rightly remarks in this connectionUnder oligopoly, advertising can become a life and death matter where a firm which fails to keep up with the advertising budget of its competitors may find its customers drifting off to rival firms. 7. Constant struggle: Competition is of unique type in an oligopolistic market. Hence, competition consists of constant struggle of rivals against rivals. 8. Lack of uniformity: Lack of uniformity in the rise of different oligopolies is another remarkable feature. 9. Small number of large firms: The numbers of firms in the market are small. But the size of each firm is big. The market share of each firm is sufficiently large to dominate the market. 10. Existence of kinked demand curve : A kinked demand curve is said to occur when there is a sudden change in the slope of the demand curve. It explains price rigidity under oligopoly.

Master of Business Administration- MBA Semester 1 MB0043 Human Resource Management - 4 Credits Assignment Set- 1 (60 Marks) Q1. Explain the need for human resource planning.

Ans:- Human resource planning system is a mandatory part of every organizations annual planning process. Every organization that plans for its business goals for the year also plan how it will go about achieving them, and therein the planning for the human resources: 1. To carry on its work, each organization needs competent staff with the necessary qualification, skills, knowledge, work experience and aptitude of work. 2. Since employees exit and organization both naturally (as a result of superannuation) and unnaturally (as a result of resignation), there is an on-going need for hiring replacement staff to augment employee exit. Otherwise work would be impacted. 3. In order to meet for the more employees due to organizational growth and expansion, this is turn call for large quantities of the same goods and services as well as new goods. This growth could be rapid or gradual depending on the nature of the business, its competitors, its position in the market and the general economy. 4. Often organization might need to replace the nature of the present workforce as a result of its changing needs, therefore the need to hire new set of employees. To meet the challenges of the changed needs of technology/product/service innovation the existing employees need to be trained or new skills sets induced into the organization. 5. Manpower planning is also needed in order to identify an organizations need to reduce its workforce. In situation where the organization is faced with severe revenue and growth limitation it might need to plan well to manage how it will workforce. Options such as redeployment and outplacement can be planned for and executed properly

Q2. What are the objectives of job evaluation? Ans:- Objectives of Job Evaluation

The decision to measure or rate jobs should only be made with the intent to reach certain objectives which are important to both management and the employee. Although there are many side benefits of job evaluation, the purpose is to work towards a solution of the many wage and compensation related administrative problems which confront the industry. The belowmentioned are some of the important objectives of a job evaluation programme: 1. Establishment of sound salary differentials between jobs differentiated on the skills required. 2. Identification and elimination of salary-related inequities. 3. Establishment of sound foundation for variable pay such as incentive and bonus. 4. Maintenance of a consistent career and employee growth policy/ guidelines. 5. In organizations with active unions, creation of a method of job classification, so that management and union officials may deal with major and fundamental wage issues during negotiations and grievance meetings. 6. Collection of job facts a) Selection of employees b) Promotion and transfer of employees c) Training of new employees d) Assignment of tasks to jobs e) Improving working conditions f) Administrative organization, and g) Work simplification.

There are many established methods to carry out job evaluation in a scientific manner. A fourfold system of classifying evaluation systems is presented here. Two are described as nonquantitative and two as quantitative. 1. Non-quantitative evaluation measures, a. The ranking system. b. The job-classification system. 2. Quantitative evaluation measures. a. The point system. b. The factor-comparison system. Q3. Why is it important to handle grievances carefully? Ans:- Importance of Grievance Handling What might happen if an organization does not provide some method by which a employee can voice his complaints and obtain a explanation? The employee will be unhappy, his productivity is impacted, he openly begins to share his discontent with not just his colleagues but also outsiders, friends, relatives, maybe even customers and vendors. Just as the employee has all the right to voice a grievance, as employer (or the management) owes it to the employee to respond suitably to the grievance. It is but commonsense that the resolution of a problem rests on management. The earliest and clearest opportunity for issue resolution is found at the first stage, before the grievance has left the jurisdiction of the manager. For this reason, many firms have specifically trained their managers on how to handle a grievance or complaint properly. If the dispute or grievance constitutes a managerial problem it can often be resolved by the manager himself with the help of the HR team. The following steps discuss how a grievance can be redressed:

1. Receiving the grievance: The manner and attitude with which the manager receives the complaint of grievance is important. The basic premise is that the manager should at the outset assume that the employee is fair in presenting his/her opinion/complaint. The complaint should not be prejudged on the basis of past experience with this or other employees. When a employee approaches the manager with a issue the manager needs to make himself available to listen it all out and provide him/her the undivided attention. Research confirms that managers who were more task-oriented, as contrasted with managers who were more people-oriented, tended to experience a significantly higher number of grievances being filed in their units. 2. Reviewing the grievance: Once a complaint is received all facts supporting the issue needs to be gathered. Proper record keeping such as performance ratings, job ratings, attending records, and suggestions are reviewed. In addition, with the increasingly legal implications of modern labour-management relations, the manager should keep records on each particular grievance. All action taken, discussions with the employee, summary and what is agreed to all of it needs to be recorded. 3. Analysis and decision: With the problem defined and the facts in hand, the manager must now analyze and evaluate them, and come to some decision. It is important for the manager to involve others in the process to ensure that it is fair and is the best solution. The manager must include the views of his own manager as he might not be aware of all the implications of the problem and its resolution. Involving HR too is a recommended process in all organizations. HR can then seek finance or legal counsel if required, before any decision is taken. All involved in the decision making process need to be aware that the decision may create an undesirable precedence within the department as well as the company. 4. Response: Often it might not be possible to provide a positive resolution to the problem. If the solution decided is adverse to the employees views, attention needs to be given to the method of communication. Employees dislike managers who will take no stand, good or bad. Clearly communicating the message and sharing as much information as possible about the decision making process helps in establishing credibility to the process used to make the decision. The manager can also invite HR or his manager to sit-in on the conversation with the employee. As far as possible this should happen in a face-to-face meeting. In the event an employee wishes to

take the appeal beyond to the next stage of the procedure he must be allowed to do so. The manager must have the opportunity to explain his decision to the other members so they can take a well-informed decision. 5. Follow up: The objective of the grievance procedure is to resolve a disagreement between an employee and the organization. Open communication is important for this process. The purpose of phase is to determine whether the employee feels that the problem has been sufficiently redressed. If follow up reveals that the case has been handled unsatisfactorily, then redefinition of the problem, further fact-finding, analysis, solution and follow up are required. At this stage the manager can step aside and allow someone else in a position of authority like the HR or the managers manager to lead the process and close it. Among the common errors of management encountered in the processing of grievances are: 1. Lack of in-depth review of the issue and its facts; 2. Expressing policy/management opinion prior to the time when all pertinent facts have been discovered; 3. Failing to maintain proper records; 4. Resorting to authoritative orders rather than having open conversations with the employee; and 5. Watch out if making exceptions / setting a precedence which could then become a rule. Q4. How can we evaluate the effectiveness of training programs conducted in organizations? Ans:- An objective of training evaluation is to determine the payoff from the training investment. It focuses on the improvement of the participant in the training programme to perform jobs for which they were trained, what was effective and what was not, whether the

trainees required any additional on the job training, and the extent of training not needed for the participants to meet job requirements. There are various approaches to training evaluation. To get a valid measure of training effectiveness, the manager should accurately assess trainees job performance two to four months after completion of training. However this focus is not easy to establish and track in most organizations. Per Kirkpatricks study, training effectiveness of outcomes can be measured: 1. Reaction: Evaluate the trainees reaction to the programme. Did he like the programme? Did he think it worthwhile? 2. Learning: Did the trainee learn the principles, skills and fact that the supervisor or the trainer worked them to learn? 3. Behaviour: Whether the trainees behaviour on the job changed because of the training programme. 4. Results: What final results have been achieved? Did he learn how to work on machine? Did scrap page costs decrease? Was turnover reduced? Are production quotas now being met? etc., Structured interviews with the immediate supervisor of the trainees are acceptable methods for obtaining feedback in training. The supervisor is asked to rate the former trainee on job proficiency directly related to the training objectives. Another approach is to involve the use of experimental and control groups. Each group is randomly selected, one to receive training (experimental) and the other not to receive training (control). The random selection helps to assure the formation of groups quite similar to each other. Measures are taken of relevant indicators of success (e.g. words typed per minute, units of work produced per hour etc.) before and after training for both groups. If the results shown by the experimental group are significantly greater than those of the control group, the training can be considered as successful. Another common method is the longitudinal or time series analysis. A series of measurements are taken before the programme begins and continues during and post completion of the

programme. The results are then plotted on a graph to ascertain changes if any, have occurred and continue to remain as a result of the training investment that was made. In addition, pre-andpost tests are administered to the training groups. Prior to the training, a test related to the training material is applied, and the results of this pre-test are compared with results on the same or similar test administered after the programme has been completed. Q5. What are the objectives of human relations? Ans:- Objectives of Human Relations A human Relations Programme thereby attempts at enhancing employee motivation and workplace morale through an improved three-way communications and through employee participation in the decision making processes. Human relations seek to emphasise employee aspects of work rather than technical or economic aspects. For example while it might be in the best interest of an organization to have a employee skilled and completely proficient in one job/ set or responsibilities, todays organization provides opportunities for employees to multi-skill and acquire knowledge of new yet related jobs/responsibilities. These acts as a motivator for employees as they benefit by learning new skills / jobs and given an opportunity can perform and excel in another job. It also seeks to make employment and working conditions less impersonal. The human relations approach emphasises policies and techniques designed to improve employee morale and job satisfaction. For example it is common place in organizations to provide for / encourage employee empowerment where-in the team brings about creative measures to reduce cost/ improve customer satisfaction. Such teams design and implement selfdriven initiatives to bring about the business result. It is believed that this is accompanied by increased employee efficiency and reduction in employee dissatisfaction. An understanding of emerging workplace human behaviour can be summarised as: i) Assist the manager to develop a better realization of how his own attitudes and behaviour play a part in everyday affairs of the team and its morale; ii) Assist the manager to develop a keener sensitivity towards the team members and interpersonal dynamics

iii) Partner with the managers in helping him drive the business goals and take part ownership of work challenges and how best to resolve them iv) Enable him to anticipate and prevent problems, or at least to resolve more effectively those that he cannot avoid; and v) Network with other teams with related dependencies and help resolve inter-team business impacting challenges The variety of causes of human relations problems lead to the conclusion that no one programme or single approach can create conditions for good human relations. Therefore, as shared earlier it in common for organizations and individuals in organization to constantly innovate and resolve challenges that will benefit both the organization as well as the employee. This helps understand the key HR objectives which can be best illustrated by understanding the functions that HR attempts to fulfil in any organization: i) Human Resource Planning estimating the need for resources in order achieve the desired business results. HR plans can be both short term/immediate as well as long term / strategic. The HR team partners with the line managers to understand the business goals and targets for the year and then together plan the HR needs in order to meet the goals. ii) Acquisition of human resources staffing the organizations with the right mix of skills and competencies at the right time. It also includes HR initiatives like promotions and internal job posting to fulfil this requirement for human resources. Staffing teams in organizations are usually a separate group of specialists who work closely with the line managers to understand the skills and competencies needed for the job and engage together to select the best talent for the open positions. iii) Training and employee development focuses on managing training activities to upgrade skills and knowledge as well as soft skills like team building and leadership. The training team is again a group of HR specialists who propose the training program and consult with the line managers to ensure that the program achieves the desired outcomes.

iv) Building performance management systems focuses on the right processes to set goals for performance as individuals/teams and related measurement methods. This is a core HR activity and is supported by the HR generalist. v) Reward systems establishing appropriate compensation systems and reward mechanisms that would reward the desired outcome and results in accordance with the corporate values. This again forms a part of the HR generalists tasks. How employees progress in a organization how they are paid w.r.t. internal and external market factors, what employee benefits are offered, are some aspect that this function redresses. vi) Human Resources Information Systems that would take care of the operational transactions from the time an employee joins till the time the employee exits, like personnel files, compensation administration, payroll, benefits administration and issuing letters and testimonials. This task is supported by as separate HR operations team who act as a HR helpdesk and provide information to the employees/managers. Q6. Assume yourself as an HR Manager. You have been given the responsibility of promoting the rightful employees. For this, performance appraisal of the employees must be carried out. What appraisal method would you choose? Justify. Ans:- The most difficult part of the performance appraisal process is to accurately and objectively measure the employee performance. Measuring the performance covers the evaluation of the main tasks completed and the accomplishments of the employee in a given time period in comparison with the goals set at the beginning of the period. Measuring also encompasses the quality of the accomplishments, the compliance with the desired standards, the costs involved and the time taken in achieving the results.

Ideally every individual in any organization needs to be appraised. The appraisal system should be able to take care of this, and it shall also meet the needs and the objectives of the organization meet. On broad way to categorise, the type of appraisal template used to assess performance, focuses on what the organization wishes to measure: Trait-based Appraisal: is used to assess personality or personal characteristics, loyalty, communication skills, level of initiative, decision making etc.. This is common in companies that are in the business of service, and customer satisfaction. While it is difficult to define the criteria accurately, rater/manager bias is high, and as the focus is on personal attributes providing feedback is a challenge. Behavior-based Appraisal: focuses on the appraisal of what an employee does on the job. It evaluates behavior and measures what an employee should do differently. This is common in customer facing organizations, like the hospitality / NGO companies. As the focus is on the assessment of behavior rater/manager bias remains a challenge here as well. Results-based Appraisal: measures objective results of work. The focus is on the outcome/deliverables from the job, and this alone serves as the measure for success. It may not be possible to use in jobs which depend on team work or where the job is service / support based. Most organizations use a combination of trait, behaviour and results in their appraisal systems as all three are important. It provides for a holistic and fair assessment of the individual. 1. Individual evaluation Methods : There exists five ways to evaluate an employee individually. Here the employee is evaluated one at a time without directly comparing him/her with other employees. 2. Multiple-person Evaluation Methods : While the above techniques are used to evaluate employees one at a time, there are a number of methods being used in organizations to evaluate an employee in comparison with other 3 . 360 Degree Appraisal : Typically, performance appraisal has been limited to a feedback process between employees and Managers. However, with the increased focus on teamwork,

employee development, and customer service, the emphasis has shifted to employee feedback from the full circle of sources depicted in the diagram below. This multiple-input approach to performance feedback is sometimes called "360-degree assessment" to connote that full circle.

Figure 7.1: The 360 degree matrix There are no prohibitions in law or regulation against using a variety of rating sources, in addition to the employees Manager, for assessing performance. Research has shown assessment approaches with multiple rating sources provide more accurate, reliable, and credible information. For this reason, HR Management supports the use of multiple rating sources as an effective method of assessing performance for formal appraisal and other evaluative and developmental purposes. The circle, or perhaps more accurately the sphere, of feedback sources consists of Managers, peers, subordinates, customers, and ones self. It is not necessary, or always appropriate, to include all of the feedback sources in a particular appraisal program. The organizational culture and mission must be considered, and the purpose of feedback will differ with each source. For example, subordinate assessments of a Managers performance can provide valuable developmental guidance, peer feedback can be the heart of excellence in teamwork, and customer service feedback focuses on the quality of the teams or agencys results. The

objectives of performance appraisal and the particular aspects of performance that are to be assessed must be established before determining which sources are appropriate. We shall discuss the contributions of each source of ratings and feedback. In addition, precautions are listed to consider when designing a performance management program that includes 360-degree assessment. 360 degree appraisals are a powerful developmental method and quite different to traditional manager-subordinate appraisals (which fulfil different purposes). As such a 360 degree process does not replace the traditional one-to-one process - it augments it, and can be used as a standalone development method. 360 degree appraisals involve the appraisee receiving feedback from people (named or anonymous) whose views are considered helpful and relevant. The feedback is typically provided on a form showing job skills/abilities/attitudinal/behavioural criteria and some sort of scoring or value judgement system. The appraisee should also assess themselves using the same feedback instrument or form. 360 degree respondents can be the appraisee's peers, up-line managers/execs, subordinate staff, team members, other staff, customers, suppliers - anyone who comes into contact with the appraisee and has opinions/views/reactions of and to the appraisee. Numerous systems and providers are available - I wouldn't recommend any in particular because my view about this process is that you should develop a process and materials for your own situation, preferably involving the appraisees in this, which like all participative approaches, often works well. You can develop your own 360 degree feedback system by running a half-day or full day workshop (depending on extent and complexity of the required process) involving the appraisees or a sample group, during which process and materials can be created and provisionally drafted. The participative workshop approach as ever will give you something that's wholly appropriate and 'owned' instead of something off-the-shelf or adapted, which would be arbitrary, mostly inappropriate and impracticable (in terms of criteria and process), and 'not invented here', ie., imposed rather than owned.

I would recommend against restricting the 360 feedback to peers and managers only - it's a waste of the potential of the 360 degree appraisal method. To use the feedback process for its fullest '360 degree' benefit involve customers (in the broadest sense - could be patients, students, users, depending on the organization), staff, suppliers, inspectors, contractors, and others for whom good working relationships and understanding with the appraisee affect overall job performance, quality, service, etc. Ensure respondents are aware of equality and discrimination issues, notably the Age Discrimination legislation and implications which might be new to some people. Comments such as 'you can't teach an old dog new tricks', or 'not old enough to command respect' are ageist, discriminatory, unlawful, and will create a liability for the originator and the employer. Developing 360 degree appraisals systems process make ideal subjects for a workshops, which in itself contains some very helpful developmental benefits and experience for all involved. If you're not able to get everyone together for a workshop you should solicit input and ideas particularly about appraisal criteria and respondents and anonymity - then draft out process and materials - then issue for approval, then pilot, review, adapt and then implement. Adapt, improve and develop on an ongoing basis. It is my view that no aspects of 360 feedback should ever be mandatory for any appraisee or respondent. Given more than three or four similar role-types being appraised it's not sensible to produce individually tailored criteria, in which case when it comes to the respondents completing the feedback not all the criteria will be applicable for all respondents, nor for all appraisees either. By the same when designing the feedback instruments (whether hard-copy documents or online materials), it's useful to allow space for several 'other' aspects that the appraisee might wish to add to the standard criteria, and space for respondents to add 'other' comments. Open honest feedback can touch sensitivities, so be sure that appraisees understand and agree to the criteria, respondents (by type, if not named) and process. Ensure suitable and sensitive counselling is provided as part of the informing of feedback results. If 360 degree feedback results are to be analysed collectively to indicate the overall/total situation (ie., to assist in determining organizational training and development needs for

instance), think carefully about the feedback form scoring system and particularly its suitability for input to some sort of analysis tool, which could be a spreadsheet, and therefore numerically based requiring numerical scores, rather than words, (words of course are more difficult to count and measure, and while words and description assessment enables more subtlety, they also allow more room for misunderstanding and misinterpretation). For guidance have a look at the skills and behavioural assessment tool - it's not a 360 degree tool, but is an example of the basis of one, and some of the skills elements that can be included in a 360 degree appraisals form. Similarly the training needs analysis tool is an example of a collective or organizational measurement tool, based on the input of a number of individual feedback assessments. This tool can easily be adapted to analyse a number of 360 degree responses.

Master of Business Administration- MBA Semester 1 MB0043 Human Resource Management - 4 Credits Assignment Set- 2 (60 Marks) Q1. State the major career development activities found in organizations. Ans:- A variety of career development activates are available for use. Some of the more popular ones include: 1. Self assessment tools:- these are usually technology enabled on-line (on the corporate intranet) tools that form part of the performance appraisal system and allow the individual to identify areas of strengths and parallelly identify career paths that would leverage these strengths the best. E.g. Career Planning Workbooks, Career Workshops hosted by the organizations from time to time. 2. Individual Counselling:- formally the process allows for individuals to discuss this as part of the performance management process with their immediate managers and share and take feedback on the appropriateness of the choices and how to go about pursuing it. Often managers recommend relevant other managers and leaders who the employee can link with to seek advice and support. Organizations also provide for formal mentoring programs to which an employee can enrol and sign up a mentor who can then provide the support and counselling on the best career option and how to go about it achieving it. 3. Information Services: organizations have established policies on what skills and experiences that each job in the organization requires. Jobs with similar skills and experiences are clubbed together to create parallel career paths. For example in a software development firm the career path options for the software engineering team can be designed as follows: Career Paths in a software company

Figure 6.1: Career Paths in a software company These are typically called career ladders or career paths and they help an employee identify what his options are for future growth and identify the appropriate one based on his personal skills and capabilities/ limitations. These career paths would be supplemented with additional information on skills and experience that one must have for each role/job in the career path. It would also specify the particular qualification or special certifications that the positions demands. An employee aspiring to pursue a career option would need to dedicate time and effort and the expenses towards acquiring the same. Large MNCs (multinational companies) also encourage the reimbursement of these expenses as a annual fixed amount on successfully clearing the exam/certification. The employee however needs to find the time and expend the effort away from work. The actual move of the employee to the new role would however depend on the existence of a job vacancy in that role. The employee can also approach career resource / talent management centres supported by the HR teams for more information on how to plan careers and apply for new roles sand jobs. Organizations usually have formal job posting systems through which the employee receives this information and applies for the job after discussing with his manager. 4. Initial employment Programs Organizations also run internship and apprenticeship programs wherein the individuals aspiring to do a particular job can spend some time as a temporary

employee to explore interest and skill fitment for the job/role. (e. g. Anticipatory socialization programs, realistic recruitment, and employee orientation program); 5. Organizational Assessment programs : organizations can proactively establish formal processes wherein an employee can volunteer to participate and understand himself/herself and his/her strengths. Through the use of Assessment Centres organizations can help an employee identify areas for improvement and means of building those skills. So he can achieve his career plans. Certain organizations offer Psychological Testing instruments which profile the employees strengths and roles and responsibilities he / she will best fit into. 6. Developmental programs focus the effort of the employee towards helping the employee to achieve his career goals. The Assessment Centres, Job rotation programs, in-house training, tuition refund plans, and mentoring, all prove effective tools to help the individual along. No matter what tools are used for career development, it is important that employees develop and individualised career plan. For example Raychem requires every person to have a learning or development plan. Q2. What are the major problems faced in benefits management? Ans:- One of the main problems faced by management in organizations is the need to maintain a competitive advantage. With the global economic climate it has been difficult for companies and organizations to keep up with their rivals, let alone better them. Corporate culture is one responsibility that management need to adhere to because it is vital when you want to succeed in business. It creates a sense of innovation and productivity as opposed to a more negative culture which may stifle employees and detrimentally affect job gratification. In addition, managers have a huge responsibility for guiding the organization in the right direction as they are the prime decision makers. Managers have to make the plans and organize their employees and resources in order to put the organization in a direction that will grant them success. Most modern management take on a strategic management style which initially states the main aim of the particular mission which follows by the processes that will be carried out on a day-to-day basis.

Moreover, one of the most important aspects that a good manager requires is solid

communication skills. It is very likely in business that you are going to come into contact with conflicting styles of communication, however it is the manager's job to be aware of this as well as their own style. They need to be able to enforce their own style of communication while being able to adapt to others, especially if they are consulting with other organisations and companies. Some of the basic problems faced by management are 1: How to produce a qualitative product: This is the first problem faced by management that what is to produce, how much to produce and where to be produce. And the organization has to decide either they have to produce different products or to emphasis on one product. 2: How to deal the labour union: The labour is the group of people working for the betterment of the employees working in the organization. The management has to decide that how to full fill the demands of the labour union in respect of salaries, bonuses,

insurance, medical allowances, fringe benefits etc. 3: How to compete in the market: Various decisions for example how to charge the price, how to place the product, how to promote the product has to be taken by the management and they try to solve these problems in a best manner. 4: How to utilize the organization resources: The management made various decisions about the organization resources that is man, money, material, machinery, market and methodology. 5: To avoid stick out situation: Stock out situation is that situation when the customer demands for the product and the company has no product at that time. The management has to decide how to tackle this problem.

Q3. What are the factors that impact recruitment in organizations? Ans:- All organizations, whether large or small, do engage in recruiting activity, though not to the same intensity. Few factors that impact the nature of recruitment: 1. The size of the organization- the smaller the organization the more the need to carefully scrutinize the candidate for a job and the fitment to the organizational culture. The risk in

case of job-candidate mismatch can prove equally expensive for a smaller organization as compared to the larger one. 2. The employment conditions in the country where the organization is located- critically impacts the recruiting strategy. The methods for recruiting, the selection tools that are most suited and the legal framework that bear on the employer are some aspects that need to be considered. 3. The affects of past recruiting efforts which show the organizations ability to locate and keep good performing people- constantly reviewing the effectiveness of the recruiting methods and the selection tools used, evaluating the success at-work of the new recruits are some methods used by organizations to ensure that quality hiring practices are inplace. 4. working conditions and salary and benefit packages offered by the organization this may influence turnover and necessitate future recruiting; (v) The rate of growth of organization- the phase in the life-cycle of the firm is a measure of the recruiting effort 5. The level of seasonality of operations and future expansion and production programmes ensuring that the recruitment numbers come form a well-planned Human Resource Plan is critical to ensure that there is no over-hiring or under-hiring of the required talent to achieve the organizational objectives. 6. Cultural, economic and legal factors these too affect the recruiting and selection methods that are used. Q4. Explain the general procedures followed in the case of a disciplinary action. Ans:- Different organizations use a variety of formal and informal methods to resolve these matters. The following steps are recommended a) An accurate statement of the disciplinary problem. b) Collection of data or facts supporting the report of the offence. c) Review policy and past similar incidents.

d) Identify corrective action to implement; take legal counsel to ensure that the local laws of the land are not in conflict e) Apply the action f) Follow-up on the disciplinary action. a. Accurate Statement of the Problem: The first step is to ascertain the problem by seeking answers to the following questions: 1. Does this case call for a disciplinary action? 2. What, exactly, is the nature of the violation or offence? 3. Under what-conditions did it occur? 4. Which individual or individuals were involved in it? 5. When, or how often, did the violation occur? In other words, an executive must first find out that a violation has occurred and that is entirely the fault, or at least partially the fault, of one or more subordinates. The next step is to determine and state the nature of the alleged violation of a rule, a regulation, a policy; to determine whether a request or order has been ignored or broken, and assess the seriousness of the specific offence which has been committed. It is also necessary to know exactly who and what was involved in the violation-whether a particular individual or group. Finally, it is desirable to know when and/or how often the violation occurred. b. Collection of data or facts support the report of the offence: Before any action is taken in a case, it is essential to gather all the facts about it. A thorough examination of the case should be made within the stipulated time limit. The facts gathered should be such as can be produced before a higher authority, if and when needed.

c. Review policy and past similar incidents: The kind of penalty to be imposed for an offence should be determined beforehand. Should it be simple reprimand, a financial or non-financial penalty? Or should it be demotion, temporary lay-off or outright discharge? d. Identify corrective action to implement; take legal counsel to ensure that the local laws of the land are not in conflict: When a decision has been taken to impose a penalty, the punishment to be awarded should be such as would prevent a recurrence of the offence. If the punishment is lighter than it should be, it may encourage the violation of the same rule or another; if it is greater than it should be, it may lead to a grievance. e. Apply the action: The application of the penalty involves a positive and assured attitude on the part of the management. "If the disciplinary action is a simple reprimand, the executive should calmly and quickly dispose of the matter. But when severe action is called for, a fortnight, serious and determined attitude is highly desirable. f. Follow-up on Disciplinary Action: The ultimate purpose of a disciplinary action is to maintain discipline, to ensure productivity, and avoid a repetition of the offence. A disciplinary action should, therefore, be evaluated in terms of its effectiveness after it has been taken. In other words, there should be a more careful supervision of the persons against whom a disciplinary action has been taken. Q5. Trace out the growth of Human Resource Management in India. Ans:- HRM in India The HRM practices in Indian organizations are a parody of sorts. There is a strong foundation that is a simple extension of the HR practices from the established western organizations and the developed countries. However there is a equally strong cultural impact on the hiring practices, compensation standards, benefits and statutory benefits, performance linked rewards and payout, which though in-step with international HRM practices have a flavour their own. We will discuss a few distinct differentiators that impact HRM practices in Indian organizations:

The Indian culture is one that is deeply rooted in its societal and collectivistic values whereby there is a natural urge to collaborate at the workplace, work in teams and groups with ease.

The diversity that is seen in the Indian society w.r.t. diverse religions and states are reflected in the ability of the Indian to accept diversity of views and thought without inhibitions. Not much effort needs to be expended to practice tolerance towards others points of view.

The tolerance for ambiguity and uncertainty however is low in the Indian which requires a lot more detailing of jobs and roles and responsibilities. There is need therefore to establish definite boundaries of authority and responsibility.

Work culture-wise, the natural ability to work hard and long hours, perseverance and the need to earn money impact the way the work is organised in Indian organizations. It is not uncommon for the Indian to compromise personal time with family / friends to instead attend to work and satisfy a customer. There is a overwhelming sense of service that manifests itself at the workplace.

Availability of a large educated group of individuals of different calibre of knowledge and skill allows the country to undertake different nature of work beginning with lower skilled activities such as BPOs(Business Process Outsourcing) and moving up the value curve to far more advanced activities .KPOs (Knowledge Process Outsourcing).

Compensation and benefits costs continue to provide a competitive advantage for India making it a preferred business location the world over. There is a natural expressed need to belong. This again is typical of the Indian and has brought about a significant amount of customisation of the workplace and its norms to allow for cultural events and activities through the year. There is a lot more scope for socialising that exists in the Indian organizations.

Current trend in the over populated urban part of the country are quickly fading out to tire 2 and tire 3 cities and town where the cost of living and wage levels are not as competitive, further expanding the Indian resource base for hiring and building talent, contributing to the overall gain of the country.

HRM in India however is not structurally well-researched and hence a lack of theoretical information to track and document its progress. Unlike in the emerged countries like the USA and the UK where the research networks are mature and well established. Research is yet at its infancy in India and there is a lot of dependence on the emerged countries in emulating best practices in the way people are hired and managed. Today the organizations of Indian origin have their unique HR strategies that work best for them. Work more and earn more is a common and accepted philosophy at the Indian workplace. Work is respected and revered and almost worshipped in Indian organizations. Benefits are basic yet equitable and adequate. There is a high focus on cash in hand as opposed to benefits and perks. Our statutory and legal frameworks, as a result, are a lot more liberal and less restrictive as compared to the emerged countries. An open society drives openness at the workplace and hypocrisy is largely discouraged, leading to well structured policies and processes that drive workplace behaviour. Diversity for India is a challenge very different from its existence in the rest of the world. Diversity initiatives in India normally focus around challenges related to gender diversity. Its only in the past few years that there is a pattern of women in the Indian corporate. Today this is a key issue. There is a lot of strategising and initiatives in all Indian MNCs as well as Indian domestic firms toward wooing this critical potential talent for organizational advantage. Q6. Assume yourself as an HR Manager of a publishing house. You find that the morale of the employees is generally low. What steps would you take to improve employee morale? Ans:- There are a number of measures which can be used to control the warning signals of low morale. The following are the positive measures to be taken to bring job satisfaction to the employees and reconcile individual interests with the interests of the organization. 1. Creation of whole jobs Under this method, complete jobs are assigned to the employees. The complexity of a job should be increased so that it may appeal to their higher needs. 2. Job enrichment Job enrichment tries to deal with dissatisfaction by increasing job depth. Under this, individual employees may be given responsibility for setting their own work pace, for concerning their own errors, and/or for deciding on the best way to perform a particular task

3. Building responsibility into a job Employees should be encouraged to participate and if possible be held responsible for taking decisions. Some delegation of responsibility from the manager tot he employee could be useful in improving employee ownership 4. Managerial effectiveness - This can be achieved by: i) Developing work groups; ii) Improving the social contacts of the employees- time away from work in team building and fun activities; iii) Managerial coaching discussed above iv) Employee stress management activities 5. Flexing working hours Flex time / work from home provisions allow employees to arrange their work hours to suit their personal needs and life-styles. This is particularly suited to situations with fluctuating workloads. Flex time employees are responsible for co-ordinating their functions with other employees and thereby have more responsibility and autonomy. 6. Rotation of jobs This reduces employees boredom which arises out of the monotonous nature of his work. 7. Incentive and Profit-sharing plans- Morale can be improved by effective incentive and profit-sharing schemes. Incentive schemes are effective in improving workplace morale. They need to carefully designed (preferably by experts) well communicated and implemented to be effective. In addition to its economic aspects, profit-sharing has also psychological aspects relating to friendly move by the management in providing the employees an opportunity to participate in the profits. Morale can also be improved by adapting several other measures such as employee contest, special recognition and awards to long service employees and training the managers in how to manage people.

You might also like